RMCS nr 16

Page 1

o. ro

© 2006, Editura „Neutrino” Titlul: Revista de matematică a elevilor şi profesorilor din judeţul Caraş-Severin I.S.S.N. 1584-9767

Societatea de Ştiinţe Matematice din România Filiala Caraş-Severin

tri n

Colectivul de redacţie:

eu

REVISTA DE MATEMATICĂ

w .n

A ELEVILOR ŞI PROFESORILOR

Bădescu Ovidiu Dragomir Adriana Dragomir Lucian Didraga Iacob Gâdea Vasilica Golopenţa Marius Moatăr Lavinia Pistrilă Ion Dumitru Stăniloiu Nicolae Şandru Marius Şuşoi Paul

DIN JUDEŢUL CARAŞ-SEVERIN

w

Nr. 16 , An VII-2006

Editura „Neutrino” Reşiţa, 2006

© 2006, Editura „Neutrino” Toate drepturile rezervate Mobil: 0724224400 www.neutrino.ro E-mail: editura@neutrino.ro 2


o. ro

CUPRINS

Gânduri

● Îţi cunoşti limitele? Cred că sunt plus şi minus infinit.

tri n

● Am o încrâncenată dorinţă: să ştiu pe dinafară tot ce şcoala nu m-a învăţat. Lupt să mi se împlinească măcar un pic.

● Nu numai matematica ne învaţă că distanţa cea mai scurtă dintre două puncte e poate drumul cel mai bun. ● Munca puţină dă mult amor propriu, în timp ce mult efort trezeşte o nemăsurată modestie.

w

w .n

eu

● Gânduri ……………………………………….………….. pag. 4 ● Note , articole ■ Despre introducerea noţiunii de şir cu asertivitate - partea a II-a (Ion Dumitru Pistrilă ) …… pag. 5 ■ Matematica şi arta (Amalia Popa ) ………….. pag. 9 ■ Aspecte metodice privind predarea inegalităţilor algebrice(Ovidiu Bădescu)......................................................pag. 11 ● Etapa judeţeană a olimpiadei 2006 …………................. pag.18 ● Concursul Interjudeţean Traian Lalescu, Ediţia a XX-a, Lugoj, 24-26 martie 2006(Lucian Dragomir).....................................pag. 21 ●Etapa Naţională a Olimpiadei de Matematică, Ediţia 57, Iaşi, 15-21 aprilie 2006(Lucian Dragomir).................................... pag.22 ●Concursul Pitagora(Ovidiu Bădescu)...................................pag.23 ● Probleme rezolvate ……………………………………....pag.26 ● Concursul revistei – ediţia a II-a (regulament , probleme propuse) ………………………….…......pag.51 ● Rubrica rezolvitorilor ………………………………….......pag.68 ● Membrii Filialei Caraş-Severin a Societăţii de Ştiinţe Matematice din România (cu cotizaţia plătită pe 2006).............................pag. 73

3

● Bucuria, durerea – nu au unitate de măsură; nici prostia. Nici înţelepciunea. Nici dragostea, mai ales. Matematica nu e chiar în toate. Şi nici nu e bine să fie. ● Probabil că un sfârşit numai atunci îţi dă mari satisfacţii: când un nou început te aşteaptă. ● Acelaşi soare ne oferă în fiecare dimineaţă alte raze. ● Probabil că trebuie să arzi o viaţă ca să străluceşti măcar câteva clipe.

● În clipele în care nu vei avea nimic deasupra capului te poţi gândi la ţigle sau la stele . Depinde de tine ce visezi! ● Să nu te pierzi niciodată, pentru că te poţi regăsi în tot şi în aproape toţi ce te-nconjoară. 4


o. ro

Despre introducerea noţiunii de şir cu asertivitate ( partea a II-a )

w

w .n

eu

(2)Şiruri mărginite şi şiruri nemărginite d) Şirul (an) n≥1 este mărginit dacă (∃) α, β∈R astfel încât α ≤ an ≤ β, (∀) n≥1. Cu alte cuvinte, un şir este mărginit dacă toţi termenii lui se află într-un interval de numere reale. Să se observe că această definiţie se apropie foarte mult de imaginea creată de termenul “mărginire” din vorbirea curentă. Exemple 1) 1, 0, 1, 0, … este mărginit căci a n ∈ [0,1], (∀)n ≥ 1. 1 1 2 ) a n = , (∀)n ≥ 1 este mărginit căci 0 ≤ ≤ 1, (∀)n ≥ 1. n n [Remarca 1] Şirurile care nu sunt mărginite se numesc nemărginite. Practic, arătăm că un şir este nemărginit folosind următorul rezultat: şirul (an) n≥1 este nemărginit dacă şi numai dacă (∀) M > 0, (∃)n0 ∈ N * astfel încât (a n 0 ) > M (acolo unde se pretează este instructiv să se demonstreze acest rezultat). Criteriul precedent este operant dacă se aminteşte şi [Proprietatea lui Arhimede] (∀) x∈R, (∃) n ∈ N, astfel încât n > x [proprietate care se demonstrează mai târziu, ( inclusiv în clasele următoare)]. [Remarca 2] Este posibil ca percepţia mărginirii unui şir să fie corectă şi totuşi elevii să nu poată preciza dacă şirul este mărginit sau nemărginit. Explicaţia acestui fapt are două componente (care acţionează separat sau împreună): Definiţia este incomodă pentru unele aplicaţii. Elevul nu este suficient de dotat şi/sau antrenat pentru a găsi numerele α şi β din definiţia d) (sau pentru a demonstra că şirul este nemărginit).

tri n

Prof. Dumitru Ion Pistrilă ,Oraviţa

În acest moment profesorul ar trebui să răspundă tehnic la întrebarea: cum ne dăm seama (noi,rezolvitorii) dacă un şir este mărginit sau nemărginit? Un răspuns de tip ingineresc s-ar putea rezuma astfel: dotarea şi/sau antrenamentul îi permit elevului să rezolve, de regulă, o asemenea problemă. Profesorul preocupat de organizarea “şantierului” ce conduce la determinarea soluţiei unei probleme, înlocuieşte răspunsul precedent cu unul ce prevede explicit posibilităţile de abordare ale acestuia. Următoarele date ar putea constitui elemente pragmatice de materializare a unor astfel de posibilităţi: Valorile termenilor sugerează mărginirea şirului (vezi exemple de la d)). Enunţul problemei este: arătaţi că şirul (an)n este mărginit. În acest caz existenţa numerelor α şi β din d) este asigurată. Trebuie “doar” să le găsim. (ghilimelele avertizează că nu este întotdeauna o problemă simplă aceasta). Aici consider că s-ar impune demonstrarea următorului rezultat: şirul (a n ) n≥1 este mărginit dacă şi numai dacă (∃) M > 0 astfel încât a n ≤ M , (∀)n ≥ 1.

5

Rezultatul este motivat acum în mintea şi sufletul elevilor căci pentru mărginire este suficient să găsească un singur număr, nu două. Accesibilitatea metodică a problemelor de mărginire se amplifică la elevi odată cu parcurgerea programei claselor IX, X, XI (funcţii mărginite, metoda inducţiei matematice, inegalităţi remarcabile etc). Nu sunt convins că înainte de sfârşitul clasei a IX-a elevii au timpul fizic necesar pentru însuşirea tehnicilor de studiu a mărginirii unui şir astfel încât să le poată aplica în contextul unor probleme netriviale (la specializarea filologie, clasa a IX-a, am reuşit să rezolv doar probleme simple de mărginire, şi acelea cu ingrediente şi poticniri). Enunţul problemei este: studiaţi mărginirea şirului (a n ) n . Rezolvarea unui asemenea enunţ presupune o anumită configuraţie a minţii căci apelează la tandemul intuiţie – demonstraţie. În această ordine de idei consider că introducerea problemelor de acest 6


o. ro

eu

f)Şirul f : N * → R, f (n) = a n , (∀)n ≥ 1 este crescător dacă

cu ad − bc ≠ 0 , a căror monotonie se studiază cu ajutorul funcţiei omografice). EPILOG Consider că la clasa a IX-a şirurile se introduc relativ lejer folosind d), iar în clasele următoare ele pot fi percepute drept cazuri particulare de funcţii. Studiul şirurilor cu ajutorul funcţiilor constituie o delicateţe a didacticii matematicii. Pentru ca şi elevii să sesizeze acest aspect sunt necesare însă două condiţii: Elevii să perceapă şirurile drept funcţii aidoma vorbitorilor de limbă străină care şi gândesc în limba respectivă. Problematica funcţiilor din programa de liceu să fie complet însuşită (în subtext se înţelege şi corect!). Faptul că specialiştii constată, aproape instantaneu, “atitudinea de funcţie” a şirului, îi determină să considere funcţiile ca gen proxim în construcţia definiţiei acestuia. Fiind conştient că formarea gândirii se realizează în etape, profesorul cu abilităţi didactice avansate îi conduce pe elevi spre zona în care şirul “cristalizează” în funcţie. Punctele de vedere din acest articol nu constituie neapărat “sentinţe didactice”. Trebuie să spun însă că aspectele tratate au pornit de la realitatea constatată atât la propriile lecţii cât şi la lecţiile ţinute de alţi colegi, lecţii la care am avut acces cu diverse prilejuri.

tri n

tip (la care, evident, şirurile nu au forma de la 1)) la clasa a IX-a constituie elemente de cascadorie didactică. Faptul că elevii foarte buni le rezolvă nu contrazice spiritul acestui articol. Întrucât am folosit mai înainte termenul “intuiţie”, îmi imaginez că ne despărţim mai uşor de acest paragraf actualizândune noţiunea: “capacitatea gândirii de a descoperi nemijlocit şi imediat adevărul pe baza experienţei şi a cunoştinţelor dobândite anterior, fără raţionamente logice preliminare” (Dicţionar de neologisme, autor FLORIN MARCU şi CONSTANT MANECA – Editura Academiei – Bucureşti 1978). f) În general, manualele şcolare nu tratează mărginirea şirurilor cu ajutorul funcţiilor (în unele manuale noţiunea se defineşte cu ajutorul mărginirii mulţimilor, dar aceasta este o altă discuţie). (3)Şiruri monotone d)Şirul (a n ) n≥1 este crescător (descrescător) dacă a n ≤ a n +1 (a n ≥ a n +1 ) (∀)n ≥ 1 .

w

w .n

(∀)n1 , n2 ∈ N * cu n1 < n 2 ⇒ f (n1 ) ≤ f (n2 ) . Se observă uşor că aici nu apare direct condiţia din d). Mai este ( f (n) < f (n + 1), (∀)n ≥ 1) ⇒ necesară demonstraţia implicaţiei: f (n1 ) < f (n1 + 1) < ... < f (n 2 ). Pe care implicaţie elevii buni “o văd”: n1 < n 2 ⇒ n1 < n1 + 1 < n1 + 2 < ... < n1 + (n2 − n1 ) = n2 ⇒ f (n1 ) < f ( n1 + 1) < ... < f ( n2 ) de unde f (n1 ) < f (n2 ) . [Remarcă ] 1) Definiţia monotoniei cu ajutorul funcţiilor reprezintă doar un aspect teoretic, fără relevanţă pragmatică. Aşa cum se cunoaşte, în problemele practice se verifică (cercetează) inegalităţile din d) (sub formele cunoscute din manuale). 2) Studiul monotoniei şirurilor cu ajutorul funcţiilor reprezintă însă (în anumite cazuri) un element al blazonului matematic (vezi cazul ax + b şirurilor definite recursiv astfel: x0 ∈ R, x n +1 = n , (∀)n ∈ N * , cx n + d 7

BIBLIOGRAFIE [1] AVĂDANEI CONSTANTIN şi colectiv – De la matematica elementară spre matematica superioară – Editura Academiei Române – Bucureşti 1987. [2] BERAR IOAN – Aptitudinea matematică la şcolari – Editura Academiei Române – Bucureşti 1991. [3] COLLIER ROBERT – Bogăţia la îndemâna tuturor – Editura Rom Direct Impex – Bucureşti 1994. [4] DINCULEANU NICOLAE, RADU EUGEN – Elemente de analiză matematică – Manual pentru clasa a XI-a – Editura didactică şi pedagogică – Bucureşti 1978. [5] Manualele alternative din ultimii 5 ani.

8


o. ro

Matematica şi arta

w

w .n

eu

Pentru “ profani”, matematica pare o ştiinţă închisă, un domeniu aproape interzis, fără prea multe conexiuni cu realitatea; aplicaţiile matematicii sunt însă multiple, ea fiind prezentă unde poate nici nu te aştepţi, inclusiv în frumosul care ne bucură sufletul şi mintea. Un concept matematic foarte des întâlnit în lumea înconjurătoare este simetria; în lucrarea “Artă şi Ştiinţă“, acad. Solomon Marcus remarca: “în opoziţie cu simetria de tip hexagonal, expresie a unui echilibru inert şi simplist, care stă la baza lumii nereale, lumea vegetală şi animală este legată de simetria pentagonală (cele mai multe flori au 5 petale, mâna omului are 5 degete, numeroase ritmuri în natura vie sunt determinate de numărul 5); această simetrie dă naştere unei periodicităţi dinamice“; jocul subtil al proporţiilor şi asimetriilor corpului omenesc, exprimat cu fidelitate prin aşa-numitul număr de aur, era la greci un model pentru formele arhitecturale. Să vedem puţin ce e cu acest număr, un număr iraţional notat de regulă cu φ (phi în limba greacă, de la Phidias, cel care a creat armonie, frumos, echilibru şi proporţionalitate a formelor în piatră). Numărul de aur este definit ca valoarea raportului dintre lungimile segmentelor în care se “taie“ un segment [AB] de către un punct C astfel încât segmentul mai mare (de exemplu [AC])să fie medie proporţională între segmentul [AB] şi partea rămasă[BC]. a b AC 2 = AB ⋅ BC sau b 2 = a ⋅ c ⇒ = = φ b c (adică numărul sau proporţia de aur); în geometrie, această “tăietură“ (aşezarea punctului C între A şi B) mai este cunoscută şi sub numele de “împărţire a unui segment în medie şi extremă raţie“. Deoarece b + c = a , înlocuind în proporţia anterioară 1+ 5 1 . obţinem 1 + = φ sau φ 2 − φ − 1 = 0 = 0 , de unde φ = φ 2

tri n

Prof. Amalia Popa , Oţelu-Roşu

E foarte posibil ca geometrii greci să fi cunoscut “tăietura de aur“ de la egipteni; deşi sub altă denumire, numărul era folosit la şcoala lui Pitagora, e amintit în “Dialoguri” de Platon, e definit de Euclid în”Elemente”, etc.; chiar dacă nu ştim cine a folosit primul formularea geometrică a problemei, cert este că egiptenii au cunoscut acest raport şi l-au folosit în construcţia piramidelor. Herodot afirma că preoţii egipteni i-au destăinuit că armonia piramidei lui Keops rezidă din faptul că aria unei feţe laterale este egală cu aria pătratului având ca latură înălţimea piramidei. Notăm VO = a , OM = c , VM = b şi astfel avem : VM ⋅ BC A(ΔVBC ) = VO 2 = ⇒ a2 2 ; pe de altă parte , a 2 + c 2 = b 2 ; din cele două egalităţi ajungem b imediat , cu notaţia x = , la c ecuaţia: x 2 − x − 1 = 0 , a cărei soluţie pozitivă este chiar φ . (Aşadar, raportul dintre apotema piramidei şi apotema bazei este chiar numărul de aur φ ). Incredibil, dar numărul nostru se regăseşte în sculptură şi arhitectură, în biologie, pictură, muzică şi economie, etc. Trebuie aici să amintim şi de şirul lui Fibonacci (a cărui idee iniţială este legată de reproducerea iepurilor) în care fiecare termen este egal cu suma celor doi termeni precedenţi; recursiv, aceasta se exprimă astfel: Fn + 2 = Fn +1 + Fn (ce este total surprinzător e că acest şir apare astăzi într-o fantastică varietate de fenomene, aparent chiar fără legătură între ele); surprinzător a fost şi este faptul că dacă primii doi termeni ai şirului sunt egali cu 1, atunci , pentru n din ce Fn +1 e tot mai apropiat de numărul în ce mai mare, raportul Fn 1,61803... , care e valoarea aproximativă a lui φ . Să mai amintim doar că Eminescu scria: “Cu mâine zilele-ţi adaugi,

9

10


o. ro

1.2.1 Inegalităţi de forma a 2 ≥ 0

Primele inegalităţi pe care elevii de gimnaziu le întâlnesc sunt de forma a 2 ≥ 0 ( adevărată pentru orice a real). Deducerea acestui rezultat se poate face de către elevi, punându-i să ridice numere reale la pătrat, atât negative cât şi pozitive. Elevii vor observa fără probleme că orice număr ridicat la pătrat este pozitiv, şi că a 2 = 0 ⇔ a = 0 . Din această relaţie se pot obţine numeroase alte inegalităţi importante. “Dacă în locul lui a punem a = x − y

eu

ASPECTE METODICE PRIVIND PREDAREA INEGALITĂŢILOR ALGEBRICE

enunţul sau şi demonstraţia inegalităţilor remarcabile? Câte inegalităţi remarcabile sunt obligatorii? Câte metode de abordare a inegalităţilor sunt absolut necesare? Binenţeles că nu se pot da reţete de succes, ceea ce urmează e doar punerea pe hârtie a experienţei proprii şi nu un exemplu neapărat de urmat şi se adresează cu precădere profesorilor care predau la clasă şi nu unor elevi superdotaţi. În sinteză, consider că trebuie neapărat atinse câteva puncte importante, acestea sunt: 1.2.1 Inegalităţi de forma a 2 ≥ 0 1.2.2. Inegalitatea mediilor 1.2.3. Inegalitatea lui Cauchy-Buniakovsky-Schwarz 1.2.4. Prezentarea mai multor metode de demonstrare a aceleiaşi inegalităţi

tri n

Cu ieri viaţa ta o scazi Şi ai cu toate astea-n faţă De-a pururi ziua cea de azi.” Notaţi acum: mâine = x n +1 , astăzi = x n , ieri = x n −1 şi strofa eminesciană se traduce în: x n +1 − x n −1 = x n , adică relaţia lui Fibonacci. Nu am reuşit aici decât să dezvăluim o infimă părticică din tezaurul de frumuseţi ascunse în adâncurile matematicii, poate însă suficient pentru a vă apleca mai mult asupra studiului şi conexiunilor sale nebănuite. (N.red.: O adevărată enciclopedie în domeniu, o excelentă şi captivantă carte, chiar şi pentru nematematicieni, a apărut anul trecut la Editura Humanitas: Mario Livio – Secţiunea de Aur, Povestea lui Phi, cel mai uimitor număr; credem că o mai găsiţi în librării sau o puteţi comanda prin Internet –sigur nu veţi regreta).

Prof.Ovidiu Bădescu , Reşiţa

w .n

Cândva demult, elev fiind, am fost izbit cu forţa unui fulger de enunţul inegalităţii mediilor, după ce abia mă familiarizasem cu mirificele notaţii de ∑ şi ∏ . Şi, ca să nu îmi mai rămână nici

w

o speranţă, am fost de-a dreptul doborât de faimoasa inegalitate a lui Cauchy-Buniakovski-Schwarz ; urma “Minkowski” , după care un refuz total de a le reţine până şi numele, darămite enunţul lor. Anii s-au scurs; ca student la Facultatea de matematică le-am reîntâlnit la fel de neprimitoare ca şi altădată. Am încercat să privesc dincolo de inabordabilitatea lor, am cerut sfatul a numeroşi profesori, însă abia după ce am ajuns să le predau atât elevilor de la clasă cât şi celor de la grupele de excelenţă, aceste inegalităţi au început să îmi devină familiare. Din păcate, am studiat numeroase culegeri şi articole în domeniu şi pot afirma cu certitudine că lipseşte o lucrare metodică necesară profesorilor la început de drum. Cum să predea, doar 11

obţinem ( x − y ) ≥ 0 ⇔ x 2 + y 2 ≥ 2 xy ”. Se pot deduce cu elevii şi 2

alte inegalităţi cerându-le lor să înlocuiască pe a cu diferite expresii. Vom fi cu siguranţă surprinşi de ideile pe care le au şi de ce nu, putem descoperi inegalităţi interesante. Se deduc sau se dau temă inegalităţi de tipul: 2 1) ( a + b ) ≤ 2 ( a 2 + b 2 ) , ∀a, b ∈ R ; 2 ) ab ≤ a 2 − ab + b 2 , ∀a, b ∈ R a+ b a+b a b ≤ , ∀a, b ∈ R+* ; 4) + ≥ 2, ∀a, b ∈ R+* 2 2 b a şi desigur lista poate continua.

3)

12


o. ro

Problema 1.2.1.2: Demonstraţi că pentru orice număr real x, numărul x8 + x + 1 este pozitiv. (G.M. 3/ 2003) Problema 1.2.1.3: Să se arate că oricare ar fi numerele reale pozitive a1 , a2 ,..., an avem inegalitatea ( a12 + 1)( a22 + 1) ...( an2 + 1) ≥ 2n

Uneori nu sunt suficiente doar calcule pentru a reduce inegalitatea la a 2 ≥ 0 , fiind necesare unele artificii de calcul, aşa cum arată exemplul următor: Exemplul 3.2.1. Arătaţi că ∀a, b, c ∈ R avem inegalitatea

impar, asta înseamnă n = 2 p, p ∈ N şi atunci a n = a 2 p = ( a 2 ) . p

Cum a2 ≥ 0, ∀a ∈ R ⇒ ( a2 ) ≥ 0, ∀p ∈ N .

w .n

p

Din nou am fi tentaţi să trecem mai departe...însă oare nu putem deduce o mică generalizare? Răspunsul salvator ar trebui să vină tot din partea elevilor, şi astfel obţinem că ∀a1 , a2 ,..., an ∈ R avem inegalitatea ( a1 ) + ( a2 ) + ... + ( an ) ≥ 0 şi 2

( a1 ) + ( a2 ) 2

2

2

2

+ ... + ( an ) = 0 ⇔ a1 = a2 = ... = an = 0 2

w

Ca şi aplicaţii se pot rezolva exerciţii de tipul: Problema 1.2.1.1: Dacă a, b ∈ R , atunci a 2 + ab + b 2 ≥ 0

13

Binenţeles, elevii sunt familiarizaţi cu definiţia mediilor din clasa a VII-a, mai mult media aritmetică este des întâlnită de fiecare dintre ei la fiecare sfârşit de semestru. Este indicat totuşi scrierea definiţiilor acestor medii pe tablă şi efectuarea unui tabel de tipul: a b 2 a+b ab a 2 + b2 1 1 2 2 + a b 6 4 9 72 13 97 13 2 2

eu

Se poate pune întrebarea firească dacă doar puterea n = 2 este cea care verifică a 2 ≥ 0, ∀a ∈ R ? Elevii sunt tentaţi să încerce cu n = 3 şi observă că pentru numere a pozitive inegalitatea este adevărată, iar pentru numere a negative, inegalitatea nu mai are loc. Să însemne că n = 2 este singura variantă? Răspunsul e NU, căci dacă n este număr natural par, inegalitatea e verificată. Am dedus o ipoteză...însă oare e adevărată, şi dacă da, cum o demonstrăm? Binenţeles că trebuie să folosim faptul că n este

1.2.2. Inegalitatea mediilor

tri n

a 2 + b 2 + c 2 ≥ ab + bc + ca Soluţie: 2 2 a + b + c 2 ≥ ab + bc + ca ⇔ a 2 + b 2 + c 2 − ab − bc − ca ≥ 0 . Pentru a putea forma pătrate perfecte, înmulţim inegalitatea cu 2 şi obţinem: 2a2 + 2b2 + 2c2 − 2ab − 2bc − 2ca ≥ 0 ⇔ ⇔ a 2 + b 2 − 2ab + b 2 + c 2 − 2bc + c 2 + a 2 − 2ca ≥ 0 2 2 2 ⇔ ( a − b ) + ( b − c ) + ( c − a ) ≥ 0 , relaţie adevărată.

Este indicat ca numerele a şi b să fie astfel alese încît ab să fie natural. Cu siguranţă elevii vor observa ordinea între rezultatele coloanelor 3, 4, 5 şi 6 şi vor deduce singuri posibilitatea ordonării numerelor. Pentru a fi siguri însă că acesta e adevărată întotdeauna, trebuie să o demonstrăm. Apare astfel teorema: TEOREMA 1.2.2: Oricare ar fi numerele reale pozitive a, b ∈ R+* avem inegalitatea: 2

≤ ab ≤

a+b a2 + b2 ≤ , ∀a, b > 0 2 2

1 1 + a b Demonstraţia e chiar la îndemâna aproape a oricui şi, oricum, se găseşte în orice carte mai serioasă de matematică aşadar nu o vom relua aici. Pentru a fi reţinută mai uşor, este indicată o interpretare geometrică. Există numeroase astfel de interpretări, însă am întâlnit doar una în care să apară toate cele patru medii.

14


o. ro

tri n

w .n

eu

Astfel, fie fie AB = a, BC = b . Fie D mijlocul segmentului a+b a+b a −b AC ⇒ AD = ⇒ BD = −a = 2 2 2 Se construieşte BE ⊥ AC şi se consideră triunghiul dreptunghic AEC , iar prin E se duce o dreaptă perpendiculară pe DE. Fie b−a BF//DE. Fie EG = .Evident FB < BE < ED < DG . 2 Din ΔAEC dreptunghic în E rezultă, folosind teorema înălţimii că EB = ab . Din ΔAEC dreptunghic în E , folosind că ED e a+b mediană ,avem : ED = . Din ΔFBE ≈ ΔBED obţinem 2 FB BE FB ab de unde obţinem FB. Deoarece nu = ⇒ = BE ED ab a + b 2 impune nimeni situarea punctului G, alegem G convenabil astfel

O primă idee ar fi considerarea a două numere pozitive, 2 şi 8 spre 2 2 16 = = , media exemplu şi atunci media lor armonică ar fi 1 1 5 5 + 2 8 8 2+8 geometrică este 2 ⋅ 8 = 4 , media aritmetică = 5 iar cea 2 4 + 64 = 34 de unde se vede imeadiat ordonarea între pătratică 2 medii. O altă idee ar fi considerarea cuvântului ar_ g _at care sugerează exact ordinea armonică, aritmetică respectiv geometrică. Se deduc sau se dau temă inegalităţi de tipul: a+ b a+b a 2 b2 1) ≤ , ∀a, b ∈ R+* ; 2) + ≥ a + b, ∀a, b ∈ R+* b a 2 2 a b 3) + ≥ 2, ∀a, b ∈ R+* care nu sunt altceva decât aplicări brute ale b a inegalităţii mediilor. Nu întotdeauna aplicarea inegalităţii mediilor este aşa de directă, aşa cum arată şi exemplul următor: Exemplul 1.2.2. Arătaţi că ∀a, b, c ∈ R avem inegalitatea

a2 + b2 b−a , şi obţinem EG = . 2 2 Reţinerea inegalităţii mediilor folosind interpretarea geometrică este totuşi destul de dificilă, şi astfel profesorul se vede nevoit să dea alte metode.

w

încât GD =

15

a 2 + b 2 + c 2 ≥ ab + bc + ca

Soluţie: Folosind inegalitatea dintre media geometrică şi a 2 + b2 b2 + c 2 c2 + a2 , prin , bc ≤ , ca ≤ media pătratică avem: ab ≤ 2 2 2 adunare obţinem inegalitatea cerută. Avem egalitate când a = b, b = c, c = a ⇔ a = b = c Se pune firesc problema generalizării, şi pentru aceasta se face analogie cu media aritmetică a mai multor numere, şi se ajunge la definiţia mediei armonice, şi a celei pătratice pentru mai multe numere. Neştiind deocamdată radicalii de ordin superior, evităm media geometrică. Pentru a cerceta dacă este adevărată inegalitatea mediilor şi în cazul a trei numere, facem un tabel asemănător celui de mai sus

16


c

3 1 1 1 + + a b c

a+b+c 3

a 2 + b2 + c2 3

Dând diferite exemple ajungem la concluzia că probabil inegalitatea mediilor este adevărată şi în cazul a trei numere, şi atunci probabil va fi adevărată în cazul a n numere pozitive. Demonstraţia acestui fapt depăşeşte nivelul de predare la clasă, însă cititorii interesaţi o pot găsi în primul capitol al acestei lucrări. Ca şi aplicaţii se pot rezolva exerciţii de tipul: Problema 1.2.2.1. Dacă a, b, c, d ∈ R şi a + b + c + d = 1 , arătaţi că: a ( b + c + d ) + b ( c + d + a) + c ( d + a + b) + d ( a + b + c) ≤ 2 (Etapa locală, Buzău, 2001) Problema 1.2.2.2. Să se demonstreze inegalitatea: y ( z + x) z ( x + y) 5 + ≤ ( x + y + z ) , ∀x, y, z ∈ R 3 3 6 (G.M. 1/ 1992)

Etapa judeţeană a olimpiadei de matematică 11. 03. 2006 , Reşiţa Comisia de concurs

Prof. Boris Vatzulik-Inspector Şcolar General Adjunct, Preşedinte Prof. Drd. Paul Şuşoi-Inspector de specialitate, Vicepreşedinte Prof. Marius Şandru, responsabil comisie gimnaziu Prof. Lucian Dragomir, responsabil comisie liceu Prof. Cristian Alin Nicola, secretar Prof. Anca Goşa, comisia de contestaţii Prof. Loreta Ciulu, comisia de contestaţii Prof. Pavel Ghimboaşă, comisia de contestaţii Prof. Mircea Iucu, comisia de contestaţii clasa a V-a Prof. Vasilica Gîdea, responsabil Prof. Claudia Buzilă, Prof. Dana Schiha, Prof. Dorina Humiţa, Prof. Mariana Iancu clasa a VI-a Prof. Lavinia Moatăr, responsabil Prof. Susana Simulescu, Prof. Sebastian Corîci, Prof. Marioara Radosavlevici, Prof. Pavel Rîncu clasa a VII-a Prof. Vasile Chiş, responsabil Prof. Maria Socol, Prof. Camelia Coandă , Prof. Carina Corîci, Prof. Marian Bădoi clasa a VIII-a Prof. Irina Avramescu, responsabil Prof. Mariana Drăghici, Prof. Lenuţa Dicu, Prof. Adriana Mara, Prof. Delia Dragomir, Prof. Mariţa Mirulescu, Prof. Janet Miuţa, Prof. Camelia Pîrvu, Prof. Radu Cocoral clasa a IX-a Prof. Iacob Didraga, responsabil Prof. Antoanela Buzescu, Prof. Otilia Bejan, Prof. Petrişor Neagoe, Prof. Nicolae Stăniloiu clasa a X-a Prof. Prof. Ion Dumitru Pistrilă, responsabil Prof. Prof. Diana Hurduzeu, Prof. Marius Golopenţa, Prof. Mircea Buzilă clasa a XI-a Prof. Stana Murg, responsabil Prof. Matei Avram, Prof. Ovidiu Bădescu, Prof. George Pascariu clasa a XII-a Prof. Rodica Iatan, responsabil Prof. Dana Mihailovici, Prof. Ciprian Călin, Prof. Iosif Găină

eu

x( y + z) + 3

o. ro

b

tri n

a

w

w .n

Bibliografie: [1] Ovidiu Bădescu – Metodica predării inegalităţilor algebrice în clasele V-IX, Lucrare de gradul I

17

18


o. ro

Nume, prenume

Cls

Şcoala

Profesor

Premiul

Popa Andreea

5

Lic.Traian Doda Caransebeş Şc.nr.6 Reşiţa Şc. Nr. 2 Reşiţa Lic.Hercules Băile Herculane Şc. Nr. 8 Reşiţa Şc. Bozovici Lic.Pedagogic CD Loga Caransebeş Grup Şc. Ind. Moldova Nouă Grup Şc. Ind. Moldova Nouă Lic. Diaconovici Tietz Reşiţa Lic.Hercules Băile Herculane Lic.Traian Doda Caransebeş Lic.Traian Doda Caransebeş Lic.Hercules Băile Herculane Şc. Nr. 2 Reşiţa Lic.Hercules Băile Herculane Şc. Nr. 2 Reşiţa Şc. Nr. 2 Reşiţa Lic.Pedagogic CD Loga Caransebeş Lic.Pedagogic CD Loga Caransebeş Şc. Nr. 3 Oţelu-Roşu Şc. Nr. 8 Caransebeş Şc. Nr. 8 Caransebeş Şc. Nr. 2 Bocşa

Dragomir Adrian

I

Unţanu G. Drăghici Mariana Golopenţa Marius

II III M

Ţunea Ana Găină Iosif Humiţa Dorina

M M I

Gîdea Vasilica

II

Gîdea Vasilica

III

Vlăduceanu Cristina Haracicu Maria

M

Dragomir Delia

M

Dragomir Delia

I

Bolbotină Constantin Şandru Marius Drăgan Vasile

II

Şandru Marius Şandru Marius Moatăr Lavinia

M M I

Moatăr Lavinia

II

Boldea Felicia Ciocan Florin Ciocan Florin Todor Veronica

III M M M

Florea Iuliana Bratosin Felix Semenescu Anca

5 5 6

Petrea Alin

6

Uţă Robert

6

Enciu Sandra

6

Minea Ela

6

Mocanu Ioana Dora Zanfir Cristian

6

Dimcea Cristian

7

Simion Larisa Gaşpar Nicoleta

7 7

Meşter Sergiu Zglimbea Diana Milcu Roxana

7 7 8

7

8

Lupu Vlad Borlovan Călin Olaru Sebastian Stăniloiu Ovidiu

8 8 8 8

w

Vlad Adina

M

Moatăr Lavinia

II

Dragomir Lucian

III

Neagoe Petrişor Golopenţa Marius

M M

Moatăr Lavinia

M

Dragomir Lucian

I

Bădescu Ovidiu Moatăr Lavinia

II III

Buzilă Mircea Murg Stana

M M

Dragomir Delia

M

Cucu Silviu Frâncu Andreea Paraschivu Andreea Secheşan Dorian Hurduzeu Ilie

Bădescu Ovidiu Mihailovici Dana Buzilă Mircea

I II III

Lic. Traian Vuia Reşiţa Lic.Traian Doda Caransebeş Lic. Traian Lalescu Lic. Traian Lalescu

Mihailovici Dana Didraga Iacob

M M

Bădescu Ovidiu Bădescu Ovidiu

M I

Lic.Traian Doda Caransebeş Lic.Traian Doda Caransebeş Lic. Bozovici Grup Şc. Ind. Moldova Nouă Lic. Traian Vuia Reşiţa

Didraga Iacob

II

Didraga Iacob

III

Găină Iosif Scorţan Gheorghe

M M

Buzilă Mircea

M

19

Unguraş Dragoş

9

Gurgu Anton

9

Dragomir Lucia

9

Popa Andreea Caraiman Gabriela Kremer Emanuela Istodor Cosmin

9 9

Popovici Doru Dochin Luminiţa

10 10

Luţă Răzvan Sava Bogdan

10 10

Iacob Alexandra

10 11 11 11

Buna Mădălina Chiş Andrei Vasile Peia Oana Tereza Andrei Corina

11 12

Bulacu Dan Pădureanu Claudia Ramona Marchiş Alexandru

12 12

eu

5 5 5

I

III M

w .n

Moţ Mihaela Pascu Andra Tabugan Dana

Dragomir Lucian

tri n

Rezultate obţinute

9

10

11 11

12 12

12

Grup Şc. Ind. OţeluRoşu Lic.Pedagogic CD Loga Caransebeş Grup Şc. Ind. OţeluRoşu Grup Şc. Ind. Anina Lic.Hercules Băile Herculane Lic.Pedagogic CD Loga Caransebeş Grup Şc. Ind. OţeluRoşu Lic. Traian Lalescu Lic.Traian Doda Caransebeş Lic. Traian Vuia Reşiţa Grup Şc. Ind. Moldova Nouă Lic.Traian Doda Caransebeş Lic. Traian Lalescu Lic. Traian Vuia Reşiţa Lic. Traian Vuia Reşiţa

20


o. ro

Etapa Naţională a Olimpiadei de Matematică Ediţia 57, Iaşi , 15-21 aprilie 2006

Trebuie să mai remarcăm aici rezultatele deosebite obţinute, în afara concursului, de către elevii Pîrvu Cătălin şi Măran Andrada de la Grupul Şcolar Ind. Moldova – Nouă (prof. Mihart Nicolae), care au obţinut punctaje corespunzătoare unei menţiuni, respectiv premiului II (clasa a IX a, respectiv a XI a) . ■

Lucian Dragomir

Zanfir Cristian – medalie de bronz Milcu Roxana – menţiune MEC şi medalie de argint Vlad Adina – medalie de bronz Popovici Doru – menţiune MEC şi medalie de argint Cucu Silviu – medalie de bronz

w

w .n

eu

Ca de fiecare dată, Lugojul s-a dovedit a fi o gazdă deosebit de primitoare; toate cadrele didactice ale Colegiului Naţional Coriolan Brediceanu, în frunte cu Domnul Director, Prof. Ing. Francisc Boldea, s-au străduit să ne facă şederea cât mai plăcută şi atmosfera de concurs cât mai apropiată de cea a marilor competiţii. Trecând peste frigul din camere, unii dintre cei mai buni elevi ai noştri au reuşit să-şi revină şi să se mobilizeze, obţinând următoarele rezultate: Zanfir Cristian - menţiune , clasa a VII-a Milcu Roxana - premiul I , clasa a VIII- a Stăniloiu Ovidiu - premiul II , clasa a VIII- a Lupu Vlad - menţiune , clasa a VIII- a Vlad Adina - menţiune , clasa a VIII -a Popovici Doru - menţiune , clasa a X- a Chiş Andrei - menţiune , clasa a XII -a Felicitări încă o dată elevilor, profesorilor care i-au pregătit, mulţumiri părinţilor care le creează condiţii de studiu, tuturor acelora care îi înţeleg pe aceşti copii de excepţie. Iar dacă anul acesta n-aţi obţinut nici un premiu, gândiţi-vă că e extraordinar că aţi ajuns şi până aici şi că nu au intrat în sac concursurile şi zilele, eforturile voastre vor fi răsplătite cândva, fiţi sigur de asta; iar dacă anul ăsta nici măcar nu aţi fost calificaţi, citiţi încă o dată rândurile de mai înainte;întotdeauna e un loc sub soare pentru fiecare, depinde însă de noi cât de călduţ este .

Unii dintre cei mai merituoşi elevi ai judeţului s-au reunit în gară la Caransebeş la începutul vacanţei, când alţii îşi făceau planuri de discoteci, plimbări, pierdut vremea, somn pe săturate, etc. După un chinuitor drum cu trenul (aproape 15 ore), am ajuns în târgul Ieşilor, cu multă oboseală şi speranţă deopotrivă. Pe scurt, cam asta a fost: deschidere, emoţii, odihnă, concurs, emoţii, plimbare Palatul Culturii, Biserica Trei Ierarhi, Casa Dosoftei, Parcul Copou, odihnă, Grădina Botanică, Teiul lui Eminescu, Bojdeuca lui Creangă, rezultate care arată bine, contestaţii, nimic, rezultate finale:

tri n

Concursul Interjudeţean Traian Lalescu , Ediţia a XX a , Lugoj , 24-26 martie 2006

Lucian Dragomir

21

Efectiv, unul din cele mai bune rezultate ale judeţului nostru din ultimii 10 ani. Nu s-a încheiat însă aventura moldovenească. Urmează Concursul interjudeţean A. Myller, simultan cu barajele pentru constituirea loturilor lărgite ele României pentru OBMJ şi OBM, OIM (olimpiade balcanice şi internaţionale); la baraje participă, de la noi, Roxana şi Doru. Aflăm între timp că Adina a câştigat o menţiune la concursul Myller, ne mai plimbăm, ne mai obosim, ne mai umplem de parfumul istoriei, culturii şi tradiţiei emanat peste tot parcă aici. În sfârşit, bucurie mare; Roxana a luat locul 9 la baraj, adică e în lotul lărgit de juniori al României (urmează săptămâni de pregătire, teste şi, cine ştie, poate …). Şi ca să fie un an cu adevărat foarte bun pentru Caraş-Severin, la festivitatea de închidere mai suntem anunţaţi că doi profesori din judeţul nostru au fost premiaţi pentru unele dintre cele mai 22


Concursul Pitagora

w

o. ro

w .n

eu

Ovidiu Bădescu “Legenda spune că Zamolxis săluşuieşte încă undeva în inima munţilor, se spune că de acolo veghează ca fiecare dintre noi să fie fericit. Încrezători în noi, am pornit către a-l întâlni iar primul pas este venerarea înaintaşilor, a lui Euclid, Zoroastru sau a lui Pitagora...” Concursul Pitagora, sună incitant şi un grup de 8 elevi împreună cu doamna profesoară Lavinia Moatar şi cu mine, Ovidiu Bădescu, am răspuns imediat invitaţiei. Aceşti elevi sunt: Nemeş Adina şi Ciobanu Oana, cls. a VI-a, Meşter Sergiu, Simion Larisa şi Zglimbea Diana, cls.a VII-a, toţi aceşti elevi de la Şc. Gen. nr.2 Reşiţa, de la C.D.Loga Caransebeş elevul Zanfir Cristian, clasa a VII-a iar Vlad Adina, Timofte Andrei, ambii elevi din cls. a VIII-a de la Lic. C.D.Loga Caransebeş. Şi, se cuvine să mulţumim domnului director al Şcolii Generale nr.2 Reşiţa, prof. Marius Şandru, căruia dragostea faţă de proprii elevi şi talentul înnăscut de bun manager au făcut posibilă deplasarea noastră la acest concurs.. Este adevărat că drumul până la Rîmnicu Vîlcea a fost greu şi obositor, că Haţeg, Petroşani, Valea Jiului şi inclusiv Tîrgu Jiu au fost pentru noi doar nişte semafoare, fără a avea timp să le vizităm, singura noastră oprire fiind la Mănăstirea Lainici. Ajunşi seara în Vîlcea, am reuşit cazarea şi cina oferită nu a fost pe placul elevilor noştri. A început căutarea de localuri şi cu greu a fost găsit ceva să-i mulţumească. Probabil foamea de premii se 23

manifesta în foame de mâncare. A doua zi concursul, concurs alcătuit din două probe: prima individuală, dimineaţa, la Vîlcea, şi cea de-a doua colectivă, la Voineasa, seara la ora 6. Nu insist asupra subiectelor de concurs, asupra oboselii acumulate şi asupra întârzierii cu care am aflat rezultatele. Din ce am discutat cu elevii, eram cu toţii destul de sceptici. Scepetici şi datorită prezenţei a 20 de judeţe şi a peste 600 de participanţi, mulţi dintre ei fiind în loturile de pregătire ale României pentru Olimpiada Internaţională. Sâmbătă seara se iveau primele zvonuri despre rezultate, însă premierea era abia duminică după-amiază, aşa că dimineaţa am mers cu toţii în miezul Pământului, la hidrocentrala Ciunget. De la intrare de unde ne-a lăsat microbuzul, am intrat într-un tunel ce cobora spre centrul Pâmăntului vreo 1, 7 km şi acolo, plin de lumini, de oameni care munceau, de zgomot asurzitor. Am fost extraordinar de fericiţi să descoperim că pe turbinele de acolo scria U.C.M.R. Reşiţa, am ajuns până la jumătate de metru de turbine, a fost un zgomot asurzitor dar a fost o experienţă unică pentru majoritatea dintre noi. Apoi, am plecat către casă şi curiozitatea rezultatelor îi măcina pe toţi elevii. Am ajuns în Sibiu, ne-am plimbat prin centrul vechi al oraşului şi acolo, în faţa camerei de filmat au primit diplomele meritate. Sunt momente în care uiţi efortul depus pentru a ajunge aici, momente care înseamnă mai mult decât orice pe lumea asta. E fericirea pe care aceşti copii o simt şi care se transmite tuturor dintre noi, este de fapt răsplata pentru tot ce înseamnă orele lungi de probleme imposibile. Ar fi multe de povestit, însă cel mai important e să le mulţumim acestor elevi pentru premiile obţinute. Astfel, la proba individuală: Nemeş Adina, menţiune, Zanfir Cristian, menţiune, Vlad Adina, menţiune, Timofte Andrei, menţiune iar la proba colectivă, Simion Larisa şi Zglimbea Diana, premiul III Vlad Adina şi Timofte Andrei, menţiune. Orice profesor ar fi fericit să aibă aceşti elevi, vă mulţumim!

tri n

frumoase probleme date la etapa judeţeană a olimpiadei: Prof. Nicolae Stăniloiu şi Prof. Lucian Dragomir. Nu mai spunem nimic despre drumul de întoarcere. Spunem doar că la anul etapa finală va fi la Piteşti, adică ceva mai aproape, iar judeţul nostru va avea ceva mai multe locuri alocate; dacă doriţi să ajungeţi acolo, trebuie să vă apucaţi din timp de lucru, să credeţi că oricare dintre voi poate reprezenta judeţul, să ştiţi că eşecurile şi succesele fac parte, deopotrivă, din frumuseţea luptelor pe care le dăm zilnic cu noi, cu cei din jur, cu cele din jur,văzute sau mai puţin văzute.

24


o. ro Probleme rezolvate din RMCS nr. 15

tri n

(problemele la care s-au primit foarte multe soluţii corecte vor avea indicate doar răspunsurile sau sugestii de rezolvare; problemele la care nu s-a primit nici o soluţie nu vor avea soluţii nici în acest număr al revistei – aşteptăm încă, acum poate chiar din partea profesorilor sau a altor pasionaţi de matematică; elevii pot trimite aşadar soluţii la orice problemă care nu are încă rezolvarea publicată. Nu trimiteţi soluţii publicate deja în revistă, nu are chiar nici un rost!).

Clasa a IV-a IV.001 Într-o operaţie de împărţire, suma dintre deîmpărţit şi împărţitor este 1022, câtul este 14, iar restul 47. Aflaţi deâmpărţitul şi împărţitorul. Instit. Elena Minea, Caransebeş Soluţie: 1022 − 47 = 975 (de 15 ori împărţitorul); 975 : 15 = 65 (acesta este împărţitorul); 65 × 14 + 47 = 957 (este deâmpărţitul). Verificare ! ■ IV.002 Patru vaci şi 36 de oi consumă zilnic 120 kg de nutreţ. O oaie consumă cu 10 kg mai puţin decât o vacă. Ce cantitate de nutreţ îi este necesară unui gospodar pentru a hrăni timp de 61 de zile două vaci şi zece oi? Instit. Elena Minea, Caransebeş Soluţie: O oaie consumă 2 kg/zi şi o vacă 12kg/zi; se ajunge imediat la totalul cerut de 2684 kg. ■

w

w .n

eu

Drumul către casă a fost mult mai vesel decât cel spre Vîlcea, fiecare dintre noi simţea că a fost un concurs util. Nu a atât de important să iei premii, e însă foarte important să descoperi că subiectele nu au fost aşa de inabordabile, că dacă erai mai atent puteai să faci mai mult, că de fapt trebuie să capeţi încredere în tine. Că, pregătirea la matematică trebuie să se facă periodic, că sunt multe concursuri la care poţi fi invitat şi că nu poţi pregăti un concurs în doar câteva zile. Am cunoscut elevi deştepţi, chiar dacă se chinuiam cu probleme de clasa a V-a, însă erau de fapt probleme de logică, probleme în care cunoştinţele de matematică nu ajutau la nimic. “Legenda spune că Zamolxis săluşuieşte încă undeva în inima munţilor, se spune că de acolo veghează ca fiecare dintre noi să fie fericit. Încrezători în noi, am pornit către a-l întâlni iar primul pas este venerarea înaintaşilor, a lui Euclid, Zoroastru sau a lui Pitagora...Se spune că întâlnindu-l simţi o fericire supremă. Am pornit pentru a-l căuta, am pornit spre miezul Pământului, am intrat în hidrocentrală. Am descoperit acolo o lume nouă, oameni care lucrau, vuiet continuu şi multă, multă lumină. Lumină care speram noi ne va ajuta să ne simţim altfel...însă nu era altceva decât o simplă lumină. Căutam altceva...eram dezamăgiţi şi aveam urme de îndoială dacă îl vom întâni vreodată. Am ajuns însă în Sibiu şi, diplomele şi cuvintele frumoase care ne-au fost adresate ne-au dat exact ceea ce căutam. Au fost clipe de fericire supremă, au fost clipe în care ne venea să strângem în braţe pe orice întâlneam în cale. Au fost telefoane date părinţilor, prietenilor, tuturor cunoscuţilor. A fost frumos, extraordinar de frumos. Ce am avut de învăţat? Faptul că nu e nevoie să cauţi fericirea, că ea vine la tine atunci când o meriţi. Că există un echilibru universal, că fiecare efort depus va fi recompensat atunci când nici nu ne aşteptăm, că merită să sacrificăm ore de internet, de club sau ore de pierdut-vremea pentru a ajunge aici. Că nu fiecare are minte pentru aşa ceva, însă că e păcat ca cei care sunt inteligenţi să nu profite de aceasta.” 25

IV.003 Un croitor foloseşte pentru confecţionarea a 4 rochii şi 4 costume 20 m de stofă, iar pentru 4 rochii şi 8 costume, 32 m de stofă. Câţi metri de stofă se folosesc pentru o rochie şi câţi pentru un costum ? Instit. Elena Minea, Caransebeş Soluţie: 2m pentru o rochie, 3m pentru un costum . ■ IV.004 Ionel îl întreabă pe bunicul său câţi ani are. Acesta îi răspunde: “Dacă voi mai trăi un sfert din ceea ce am trăit şi încă 5 ani, atunci voi avea 85 de ani.” Aflaţi câţi ani are bunicul . Instit. Elena Minea, Caransebeş Soluţie: 64 de ani . ■

26


o. ro

tri n

IV.010 Într-o clasă sunt de două ori mai multe fete decât băieţi. Pot fi 25 de elevi în clasă? Dar 24? Care din următoarele numere poate fi numărul elevilor din clasă: 17 , 18 , 19 , … , 32? (Încercaţi o soluţie cât mai scurtă a probemei) . Înv.Elena-Stanca Codilă, Caransebeş Soluţie: Dacă numărul băieţilor este x , al fetelor va fi 2x, deci în clasă avem 3x elevi (număr multiplu de 3). Putem avea deci 18, 21, 24, 27 sau 30 de elevi. ■ Clasa a V-a V.024 La o împărţire cu rest, mărind deâmparţitul cu 91, câtul a crescut cu 7. a) Aflaţi deâmpărţitul; b) Care sunt deâmpărţitorii pentru câtul egal cu 154 ? Prof. Ion Belci, Reşiţa Soluţie: a) Fie d=î⋅c+r, r<î (conform teoremei împărţirii cu rest) Mărim pe d cu 91 şi pe c cu 7 ⇒ d+91=î⋅(c+7)+r ⇒d+91=î⋅c+7⋅î+r deci 91=7⋅î ⇒ î=13; b) c=154, î=13, r<13⇒r∈{0,1,2,...,12} ⇒ d∈{2002, 2003,2004,…, 2014} pentru ca d1=13⋅154=2002, d2=13⋅154+1=2003,…, d13=13⋅154+12=2014. ■ V.025 Determinaţi numerele naturale ab scrise în baza 10 pentru care sunt îndeplinite simultan condiţiile: 1) a b + b a este divizibil cu 5;

w .n

eu

IV.005 Calculaţi a + b + c + d + e , dacă: 1) a este cu 392 mai mare decât b; 2) b este de 5 ori mai mic decât sfertul lui c; 3) c este de 4 ori mai mare decât jumătatea lui d; 4) d este dublul lui e; 5) 4560 este triplul lui e . Instit. Elena Minea, Caransebeş Soluţie: e = 1520, d = 3040, c = 6080, b = 304, a = 696, S = 11640 .■ IV.006 Suma dintre un număr, jumătatea sa şi treimea sa este 660. Care este numărul? Instit. Mirela Tătar, Caransebeş Soluţie: 360. ■ IV.007 Tavi a citit în vacanţă o carte. Verişoara sa l-a întrebat câte pagini are cartea şi Tavi i-a spus: “ Dacă mai avea un sfert din numărul de pagini şi încâ 50, cartea ar fi avut 550 de pagini, dar nu are aşa de multe”. Verişoara, care ştie ceva aritmetică, a scris ceva pe o foaie şi i-a spus: “Sigur ai citit mai puţin de 432 de pagini”. Ştie aritmetică verişoara? Instit. Mirela Tătar, Caransebeş Soluţie: (550 − 50) : 5 = 100 pagini ( are un sfert din carte ) ⇒ Cartea are 400 de pagini, aşadar verişoara ştie aritmetică. ■ IV.008 Tavi (acelaşi) colecţionează timbre şi l-a convins pe prietenul său Sebi să încerce şi el. Împreună, colecţia lor are acum 238 de timbre. Din toate acestea, Tavi îi face cadou prietenului 24 de timbre şi astfel Sebi are acum de şase ori mai puţine timbre decât Tavi. Câte timbre ar avea Sebi , dacă Tavi i-ar mai da jumătate din timbrele sale? Câte timbre avea Tavi la început? Instit. Mirela Tătar, Caransebeş Soluţie: Sebi are 238 : 7 = 34 de timbre , deci Tavi are 204; Jumătate

din timbrele sale reprezintă 102 ⇒ Sebi ar avea 102 + 34 = 136 de timbre. Tavi avea iniţial 228 timbre. ■ IV.009 Câte zile sunt într-o perioadă care începe şi se sfârşeşte cu un an bisect? Înv.Rodica Putiţincuş, Caransebeş Soluţie: anul bisect are 366 de zile faţă de cele 365 ale unuia nebisect;

w

avem astfel: an bisect + an + an + an + an bisect ⇒ 2 × 366 + 3 × 365 = 1827 zile. ■

27

2 ) ba − ab este cub perfect.

Prof.Delia Marinca, Timişoara Soluţie: ba − ab = 9(a − b) ⇒ b – a = 3; analizăm cazurile posibile şi ajungem la numerele : 14, 36, 47. ■ V.026 Reconstituiţi adunarea:

(COD + COD) + (COD + COD) = TON .

(Sperăm că vă place să mâncaţi peşte; dacă nu, educaţi-vă, e foarte sănătos. Lăsând gluma la o parte, probabil că ştiţi că literelor diferite le corespund cifre diferite şi literelor identice le corespund aceleaşi cifre). *** Soluţie: Se obţine imediat că C = 1 sau C = 2 (altfel suma din stânga conduce la un număr de peste 3 cifre); se ajunge la 5 soluţii ale problemei (prea puţini aţi găsit atâtea !): {102, 408},{201, 804},{134, 536}, {234, 936}, {198, 792}. ■

28


o. ro

Soluţie: A = {1,3, x} ; folosind ( d ) avem că:

V.027 O bunică are doi nepoţi. Vârsta bunicii este un număr format din două cifre astfel încât prima cifră indică vârsta unui nepot, iar a doua cifră vârsta celuilalt. Ce vârste au fiecare dacă suma vârstelor lor este 69 de ani? Concurs Bulgaria Soluţie:Bunica are 57 de ani, iar nepoţii 5, respectiv 7 ani. (Într-adevăr avem : xy + x + y = 69 ⇒ 11x + 2 y = 69 ; deducem

a ≠ b !! ) . Dacă x ≥ 4 avem că suma elementelor lui B depăşeşte 25 ⇒ x = 2 (folosim şi (a)), deci : A = {1,2,3} şi B = {1,2, 3 , 4 , 6 , 9 }■ V.032 Determinaţi produsul a ⋅ b ⋅ c ⋅ d dacă

a + c = b + d şi abba + bab = bacd .

tri n

Prof.Adriana Dragomir, Oţelu-Roşu Soluţie: 1680. ■ V.033 Două numere naturale de câte cel mult şase cifre sunt scrise numai cu cifrele 1, 4, 6 şi 9. Poate fi unul dintre numere de 17 ori mai mare decât celălalt ? *** Soluţie: Dacă unul dintre numere este de 17 ori mai mare decât celălalt , ultima cifră a unuia va fi 7, 8, 2 sau 3 care nu este printre cele patru date aşadar răspunsul este negativ. ■ Clasa a VI-a VI.024 Determinaţi numărul natural par n ştiind că suma divizorilor săi este 1+n . *** Soluţie: Dacă n nu este număr prim, atunci are şi alţi divizori în afara

w .n

eu

imediat că x ∈ {4, 5, 6} şi analizăm cazurile ce apar). ■ V.028 Un tren parcurge distanţa de 60 km dintre Caransebeş şi Lugoj într-o oră. În acelaşi timp cu trenul, o rândunică porneşte în zbor din Caransebeş spre Lugoj cu o viteză de 90 km/oră şi ajunge mai devreme decât trenul la destinaţie, apoi se întoarce zburând spre tren până la întâlnirea cu el, apoi din nou spre Lugoj; rândunica îşi continuă astfel zborul, până când ajunge odată cu trenul la Lugoj. Ce distanţă a parcurs rândunica efectuând aceste zboruri ? *** Soluţie: Evitaţi calculele inutile! Problema e mai mult de atenţie şi perspicacitate: rândunica zboară timp de o oră, deci parcurge 90 km.■ V.029 Tatăl şi fiul au de făcut o lucrare în 3 zile.Tatăl lucrează de 3 ori mai repede decât fiul său. Dacă lucrează numai fiul, în câte zile termină lucrarea? Prof.Mariana Zălog, R.M.T Soluţie: 12 zile. ■ V.030 Se dau numerele naturale consecutive a şi b (a > b) . ( i ) Care este restul împărţirii lui a la b ?; ( ii ) Care este restul împărţirii lui b la a ? . (Încercaţi, eventual, pe un exemplu concret) Justificaţi toate răspunsurile. Gazeta Matematică

1 ⋅ 1 = 1,1 ⋅ 3 = 3,3 ⋅ 3 = 9, x,3 x, x 2 ∈ B (condiţia d nu impune a ,b ∈ A ,

Soluţie: ( i )Dacă b = 1 restul este evident 0; dacă b ≥2, restul este 1; ( ii ) b = a ⋅ 0 + b , deci restul este b. ■ V.031 Determinaţi mulţimile A şi B de numere naturale care satisfac proprietăţile: a ) A are trei elemente;

w

b ) 1 ∈ A şi 3 ∈ A; c ) suma elementelor lui B este egală cu 25; d ) pentru orice a, b ∈ A , avem a ⋅ b ∈ B. Prof.Adriana Dragomir, Oţelu-Roşu

29

lui n şi a lui 1, deci suma divizorilor depăşeşte n + 1 ⇒ n este prim; fiind

par ⇒ n = 2. ■ VI.025 Pe laturile (AB) şi (AC) ale triunghiului ABC se construiesc în exterior triunghiurile echilaterale ABX şi ACY. Notăm cu P, Q, R respectiv mijloacele segmentelor [AX], [ AY ] şi [ BC ]. Este triunghiul PQR echilateral ? Concurs India ( RMT ) Soluţie în RMT 1/2002. ■ VI.026 Într-un bloc sunt 88 de apartamente cu două şi respectiv cu trei camere. Câte apartamente de fiecare fel sunt dacă numărul camerelor este 198? Prof.Nicolae Ivăşchescu, Craiova – Gazeta Matematică Soluţie : 22 apartamente cu 3 camere, 66 cu 2 camere. ■ VI.027 Fie [OA bisectoarea unghiului EOD, semidreapta [OD este opusa semidreptei [OB. Ştiind că A şi C se găsesc în semiplane diferite faţă de dreapta BD, OC⊥OA şi că m(∠COD)=47044’23”, a) Calculati m(∠EOB); b) Dacă (OF ⊂ Int(∠BOE), m(∠EOF) este cu 5028’46” mai

mare decât m(∠BOA), aflaţi m(∠BOF). Profesor Ion Belci, Reşiţa

30


o. ro

VI.031 Fie ABCD un romb de latură 1. Pe laturile (AB) şi (AD) există punctele P, respectiv Q astfel încât perimetrul triunghiului APQ este 2 şi

m(∠PCQ) =

1 m(∠BCD ) .Determinaţi unghiurile rombului dat. 2

tri n

Cristinel Mortici, R.M.T. Soluţie: Căutaţi în RMT nr. 4/2003. ■ VI.032 O grădină de formă dreptunghiulară are lungimea x metri şi lăţimea y metri (x , y numere întregi). Determinaţi x şi y ştiind că grădina poate fi încercuită cu o alee de lăţime un metru şi astfel încât aria aleii să fie egală cu aria grădinii. Concurs Bulgaria Soluţie: Există două posibilităţi: x = 10 , y = 3 şi x = 6 , y = 4 (prea puţini dintre elevi au sesizat acest lucru) . ■ VI.033 La plecarea în vacanţă 7 elevi au decis ca fiecare dintre ei să trimită exact la 3 colegi câte o scrisoare fiecăruia. Este posibil ca fiecare elev să primească scrisori de la toţi colegii cărora el le-a scris ? Concurs Moldova Soluţie: Presupunem că fiecare elev primeşte cărţi poştale de la colegii cărora le-a scris. În acest caz , oricare doi colegi A şi B îşi trimit , între ei ori două cărţi poştale, ori nici una. Astfel, numarul de cărţi poştale trimise de cei 7 elevi este par, însă 7 ⋅ 3 = 21 este impar, contradicţie. Răspunsul este deci negativ. ■

w .n

eu

Soluţie: a) Din OA⊥OC ⇒ m(∠AOD) = 900 − 47044’23” = =42015’37”. [OA bisectoarea ∠EOD⇒ m(∠EOA) = m(∠AOD) = 42015’37”. m(∠EOB)=1800−m(∠EOD)şi m(∠EOD) = 2⋅(42015’37”)= 84031’14” ⇒ m(∠EOB) =1800−84031’14”=95028’46”; b ) enunţ greşit ( redacţia îşi cere scuze, dar responsabilitatea o poartă autorul. Fiţi înţelegători , oricui i se poate întâmpla ) . ■ VI.028 Împărţiţi în trei unghiuri congruente un unghi de măsură 1080 folosind numai rigla şi compasul . Prof.I.Nicolaescu – Gazeta Matematică Soluţie: Considerăm unghiul dat ∠AOB de măsură 108 0 şi construim cu rigla semidreapta (OC opusă lui (OA , aşadar ∠BOC are măsura 72 0; e suficient să construim bisectoarea acestui unghi şi vom obţine un unghi de măsură 36 0, adică o treime din unghiul dat. Cum construim deci bisectoarea unui unghi BOC dat? Punem vârful compasului în O cu o deschidere oarecare şi trasăm un arc de cerc care intersectează OB şi OC în M, respectiv N. Punem vârful compasului în M, apoi în N şi, cu aceeaşi deschidere (suficient de mare, dar nu prea mare !) trasăm două arce de cerc care se intersectează în P ⇒ ΔPON ≡ ΔPOB . Avem imediat că [OP este bisectoarea lui ∠BOC , deci am obţinut un unghi de măsură 36 0: ∠POC . Continuaţi. ■ VI.029 Se poate împărţi numărul 2006 în părţi (numere naturale) direct proporţionale cu numerele 0,4 , 0,6 şi 0,7 ? Prof. Adriana Dragomir, Oţelu-Roşu Soluţie : 472, 708, 826. ■ VI.030 În triunghiul ABC isoscel avem m(∠BCA) = 100 0 . Bisectoarea unghiului ∠CAB intersectează pe BC în punctul D. Demonstraţi că există m ∈ ℕ astfel încât: m ⋅ AB = AD + CD .

Concurs Bulgaria Soluţie: Fie E un punct pe AB aşa încât AE = ED; deoarece m(∠DAB) = 20 0 avem: m( ADE ) = m( AED) = 80 0 ; din

m( DBE ) = 40 0 ajungem la: m( BDE ) = 180 0 − (100 0 + 40 0 ) = 40 0 deci

w

DBE este triunghi isoscel cu BE = DE; notăm cu P şi Q picioarele perpendicularelor din D pe AB, respectiv AC. Deoarece AD este bisectoare avem DP = DQ ⇒ ΔDPE ≡ ΔDQC ( ULU);ajungem astfel la: CD = DE = BE , de unde AB = AE + BE = AD + CD ⇒m=1. ■

31

Clasa a VII-a VII.024 Dacă numerele naturale a, b, c sunt direct proporţionale cu 11, 9, respectiv 7, iar numerele naturale c, d, e, f sunt invers proporţionale cu numerele 15, 21, 35, respectiv 105, demonstraţi că: 2

3

4

⎛ f ⎞ ⎛ f ⎞ ⎛ f ⎞ ⎛ f ⎞ ⎛ f ⎟ + ⎜⎜ ⎟ +⎜ ⎟ +⎜ ⎜ ⎟+⎜ ⎝a −b⎠ ⎝b−c⎠ ⎝c−d ⎠ ⎝d −e⎠ ⎝e− f

5

⎞ ⎟⎟ < 1 . ⎠

Prof. Delia Marinca, Timişoara Soluţie:

a b c = = (1); c = d = e = f ⇔ c = d = e = f (2) ⇒ 11 9 7 1 1 1 1 7 5 3 1

15 21 35 105 a b c d e f = = = = = = k ⇒a=11k, b=9k, c=7k, d=5k, e=3k, f=k 11 9 7 5 3 1

32


devine: 2 + 2 + 52 + 2 + 1 = 2 5− 1 < 1 . ■ 3

2

5

2

2

VII.025 Fie patrulaterul inscriptibil ABCD, I intersecţia diagonalelor sale şi punctul V astfel încât I este centrul cercului înscris în triunghiul VAB. Să se demonstreze că VI ⊥ CD . Prof. Petrişor Neagoe, Anina Soluţie: I – centrul cercului înscris în ΔVAB şi [AC este bisectoarea ∠VAB şi [BD este bisectoarea ∠VBA . V D Fie {A1} = VA ∩ CD şi {A 2} = VB ∩ CD A1

}

A2

ABCD - inscriptibil ⇒ )BDC ≡ )BAC ⇒ [AC - bisectoarea )VAB ⇒ )VAC ≡ )BAC ⇒ )BDC ≡ )VAC (1)

B A

Luăm punctul S∈AC astfel ca MS=MC. Deoarece BM⊥SC avem că ∆BCS este isoscel cu vârful în B. Deci ∆ABC şi BSC sunt asemenea deoarece sunt isoscele cu unghiurile de la bază congruente (∠ACB este

SC BC BC 2 = ⇒ AC = (2). BC AC 2 MC Înlocuim (2) in (1) şi ⇒ BC 3 = 2 MC ⋅ CE ( AB + BC ). ■

comun) ⇒

C

Din (1) şi (2) ⇒ m()A2DB)+m()A2BD)= m()A1AC)+m()A1CA) ⇒

}

⇒+VA1A2 - isoscel ⇒ VI ⊥ CD ■ [VI - bisectoarea )AVB VII.026 a ) Arătaţi că există o infinitate de numere iraţionale a, b pentru care a + b este raţional; ⇒ m()VA2A1)=m()VA1A2) ⇒

A − 2 B este raţional. 2

)

w .n

(

b ) Dacă 1 + 2 2

2006

= A + B 2 , cu A, B ∈ ℚ , arătaţi că

Prof. Ecaterina Zsibriczki, Bocşa Soluţie: a) De exemplu: a = x + 2 , b = x − 2 ,cu x ∈ ℤ;

(

)

2006

w

= A − B 2 şi A 2 − 2 B 2 = ( A + B 2 )( A − B 2 ) .■ b ) 1− 2 VII.027 Într-un ∆ABC cu AB=AC se notează cu E şi M punctele de intersecţie ale bisectoarei respectiv înălţimi din B, cu latura AC. Să se demonstreze că BC 3 = 2 MC ⋅ CE ( AB + BC ). Prof. Anghel Păun, Reşiţa 33

AB AE AB + BC AC AC ⋅ BC ⇒ , (1). = = ⇒ EC = BC EC BC EC AB + BC

VII.028 Se consideră mulţimea A={1, 2,…,10}. Determinaţi mulţimile B şi C care satisfac: a) B ∪ C = A ; b) B ∩ C = ∅; c) B are două elemente; d) produsul elementelor din B este egal cu suma elementelor din C. ***

Soluţie: B={6, 7}, C = A ∖ B. ■ VII.029 Fie un unghi cu măsura de 300 cu vârful în O. Pe una din laturile unghiului se ia punctul A1, diferit de vârf. Piciorul perpendicularei din Ai, i=1, 2, …, n, pe cealaltă latură este Bi, piciorul perpendicularei din Bi pe OA1 este Ai+1. a) Să se găsească i astfel încât OAi ≤ AiA1; b) Cât reprezintă AiA1 din OA1 ? Prof. Ion Belci, Reşiţa

eu

}

ABCD - inscriptibil ⇒ )ACD ≡ )ABD ⇒ [BD - bisectoarea )VBA ⇒ )VBD ≡ )ABD ⇒ )ACD ≡ )VBD (2)

I

tri n

4

o. ro

Soluţie : Aplicăm teorema bisectoarei:

Înlocuim în membrul stîng al inegalităţii cerute şi acesta

Soluţie: i=4 si A4A1=

37 OA1. 64

VII.030 a) Arătaţi că există numere întregi a, b, c astfel încât a 2 + b 2 − 8c = 9 ; b)Arătaţi că nu există numere întregi a, b, c astfel încât a 2 + b 2 − 8c = 6 . Concurs Bulgaria, Chile Soluţie: a) E suficient să dăm un exemplu: c = 1, b = 1, a = 4;

b) a 2 + b 2 = 8c + 6 ⇒ a 2 + b 2 este număr par, deci a şi b au aceeaşi paritate; dacă sunt pare , a 2 + b 2 se divide prin 4, dar 8c+6 nu, iar dacă a, b sunt impare se ajunge la a 2 + b 2 = 8l + 2 ≠ 8c + 6 . ■

34


o. ro

VII.031 În triunghiul oarecare ABC, fie M mijlocul laturii [BC],

Clasa a VIII-a VIII.024 Dacă a, b, c sunt numere strict pozitive care au suma egală cu 3 , arătaţi că: 1 + a + 2 + b + c + 3 < 6 . Prof. Adriana Dragomir, Oţelu-Roşu Soluţie: Folosim inegalitatea mediilor:

E ∈ (AC), F ∈ (AB) astfel încât [ME este bisectoarea unghiului AMC. Demonstraţi că [MF este bisectoarea unghiului AMB dacă şi numai dacă EF este paralelă cu BC. Prof. D. M. Bătineţu-Giurgiu, Bucureşti Soluţie: Cu teorema bisectoarei avem

1+1+ a (inegalitatea este strictă pentru că în 2 condiţiile ipotezei avem 1 ≠ 1 + a ); procedăm la fel în continuare. ■

AE AM = ; EF // BC dacă şi EC MC

AE AF sau, combinând cu proporţia anterioară,: = EC FB AM AF AM = = (deoarece M este mijloc) ⇔ [MF este bisectoare. ■ MC FB MB

1 + a = 1 ⋅ (1 + a ) <

VII.032 În paralelogramul ABCD punctul E este mijlocul laturii (AD). Demonstraţi că dacă F este piciorul perpendicularei din B pe CE, atunci triunghiul ABF este isoscel. Concurs Moldova

w .n

eu

Soluţie: Considerăm M astfel încât CE = EM ⇒ DCAM paralelogram şi avem imediat M, A, B coliniare; în triunghiul MFB avem FA mediană, deci FA = AM = AB. Concluzia e imediată. ■ VII.033 Iepuraşul va împărţi un număr de bomboane la 13 băieţi şi 10 fete astfel încât fiecare dintre ei să primească cel puţin o bomboană.Fiecare fată şi fiecare băiat va primi acelaşi număr de bomboane.Copii au constatat că există un singur mod de a împărţi în acest fel bomboanele.Care e numărul maxim de bomboane pe care îl va avea iepuraşul ? Concurs Moldova Soluţie: Notăm cu x numărul total de bomboane şi cu a, respectiv b numarul de bomboane primite de un băiat, respectiv o fată ⇒ 13a + 10b = x (adică am obţinut o ecuaţie diofantică – citiţi numarul anterior al revistei noastre!) Se ajunge astfel la: a = −3 x + 10k şi

VIII.025 În paralelipipedul dreptunghic ABCDA’B’C’D’, A1 este intersecţia dreptei AA’ cu semidreapta cu originea în B care face ca ∠ABA’ ≡ ∠ABA1, AP mediană în ∆ABA1, iar Q intersecţia dreptelor AD cu A1D’. Aflaţi volumul poliedrului APQA’BD’ în cazul când: a) AB≡AD≡AA’=a b) AA’=a; AB=b;AD=c. Prof. Ion Belci, Reşiţa Soluţie: În ∆A’A1B, AB este înălţime şi bisectoare ⇒∆A’A1B este isoscel cu A1B≡ A’B ⇒AA’≡AA1(*)⇒mediana AP este linie mijlocie în ∆A’A1B⇒AP|| A’B şi AP= A’B/2. AD∩ D’A1={Q}⇒AQ|| A’D’, (*)⇒AQ linie mijlocie în ∆A1A’D’⇒AQ= A’D’/2 ⇒PQ linie mijlocie în ∆A1BD’⇒PQ=BD’/2⇒ (APQ)||( A’BD’)⇒APQ A’BD’ este trunchi de piramida. a) AA’≡AB≡AA1⇒m(∠ A’BA1)=900. Avem A1B⊥ BA’, BA’⊥ A’D’,

tri n

numai dacă

b = 4 x − 13k , k ∈ ℤ;cum a, b > 0 avem imediat:

3x 4x <k< . Dacă 10 13

w

4 x 3x x x ∈ℕ, − = > 2 , adică x > 260, se ajunge la două soluţii 13 10 130 130 k1 , k 2 (contradicţie cu observaţia copiilor!). Pentru x = 260 avem imediat

k = 79 şi apoi a = 10, b = 13, deci valoarea maximă cerută este 260. (ştiau ceva matematică copii, dar şi iepuraşul!). ■

35

RT 3⊥

A’A1⊥ A’D’ ⇒ A1B⊥( A’BD’)⇒ A1B⊥(APQ)⇒PB înălţimea

trunchiului de piramidă, PB=A1B/2.

a 2 a 3 ; BD’= a 3 ; PQ= ; 2 2 a a2 2 , AQ= . A’B⊥ A’D’⇒SA’BD’= A’B⋅A’D’/2, SA’BD’= 2 2 a2 2 a 2 7 3 , PB= A1B/2= A’B/2= . Se obţine V= SAPQ= a . 8 2 24 Din datele problemei ⇒ A’B= a 2 ; AP=

RT 3⊥

b) Fie AR⊥A’B, RA’⊥ A’D’, AA’⊥ A’D’ ⇒ AR⊥(A’BD’)⇒AR

36


o. ro

înălţimea trunchiului de piramidă. AR⋅A’B=AA’⋅AB ⇒ AR =

AA'⋅ AB A' B

În ∆DAP, m(∠DPA)=900, m(∠PAD)=600⇒AP=9 cm, PD= 9 3 cm. PD⊥(ABC), BP⊂(ABC)⇒BP⊥PD⇒ d(B,PD)=BP. TP

AA’=a; AB=b; AD=c ⇒A’B= a 2 + b 2 şi BD’= a 2 + b 2

În ∆BAP, m(∠A)=900 ⇒ BP= 3 73 cm.

c a2 + b2 c a 2 + b2 AR= ; AP= ; AQ= ⇒ SA’BD’= , 2 2 2 a2 + b2 a ⋅b

(

mijlocul lui [AC]. Fie AN⊥BC si PN⊥BC⇒PM||AN⇒PM linie mijlocie în TC

)

7 AR S A' BD ' + S APQ + S A' BD ' ⋅ S APQ = abc. ■ 24 3 VIII.026 Determinaţi câte triplete (x , y , z) de numere naturale cu x < y există astfel încât numărul n = abx ⋅ aby + z 2 este pătrat perfect . Prof.Delia Marinca, Timişoara Soluţie: Dacă x<y, fie u ∈ N, 0<u≤9 astfel încât y=x+u. Astfel, n = (10 ab +x)(10 ab +x+u)+z2 Notăm 10 ab +x=t şi obţinem: n=t(t+1)+z2=t2+tu+z2, de unde deducem că n este pătrat perfect pentru u=2z (u=-2z nu convine), prin urmare y=x+2z. Dar y este cifră şi x≠y, rezultă că z ∈ {1, 2, 3, 4} şi vom avea următoarele cazuri: 1. Dacă z=1, y=x+2, x ∈ {0, 1, 2, …, 7} 2. Dacă z=2, y=x+4, x ∈ {0, 1, 2, …, 5} 3. Dacă z=3, y=x+6, x ∈ {0, 1, 2, 3} 4. Dacă z=4, y=x+8, x ∈ {0,1} Tripletele cerute sunt:(x, x+2, 1), x ∈ {0, 1, …, 7}; (x, x+4, 2), x ∈ {0, 1, …, 5};(x, x+6, 3), x ∈ {0, 1, 2, 3}şi (x, x+8, 4), x ∈ {0, 1}. ■ VIII.027 Pe planul ∆ABC cu m(∠BAC)=900 se ridică perpendiculara DP, P∈(AC). Ştiind că AB=24 cm, BD=30 cm şi m(∠PAD)=600, calculaţi distanţa de la B la PD. Dacă BC=30 cm, calculaţi distanţa de la A la (BDC). Profesor Ion Belci, Reşiţa

BN=19,2cm ; AB⋅AC=AN⋅BC⇒AN=14,4cm⇒PM=7,2cm. T 3⊥

w .n T 3⊥

w

Soluţie: DP⊥(ABC), PA⊥AB ⇒ DA⊥AB. În ∆DAB, m(∠DAB)=900 ⇒ DA=18 cm.

37

TP

PD⊥(ABC), PM⊥BC ⇒ DM⊥BC. În ∆DPM, m(∠P)=900 ⇒

DM2=PD2+PM2⇒ DM=

9 91 cm. (DPM)⊥(DBC) . Fie NQ||MD, 5

Q∈BD, PN||MP⇒(ANQ)||(DMP) ⇒ (ANQ)⊥(DBC) si (ANQ)∩(DBC)=NQ. În ∆ANQ fie AT⊥NQ⇒AT⊥(BCD)⇒

eu

V=

∆CAN ⇒ AC2=BC⋅NC⇒NC=108/10=10,8cm ⇒ NM=MC=5,4cm,

tri n

SAPQ=

a2 + b2 c ⋅ 2 2 2 2 = c a +b . ⇒ 2 8

TP

În ∆ABC, m(∠BAC)=900, BC=30cm, AB=24cm ⇒ AC=18 cm⇒P-

d(A,BCD)=AT ⇒ AT=

144 273 cm. ■ 91

VIII.028 Laboratorul de biologie are forma unui paralelipiped dreptunghic cu dimensiunile de 8m, 8m şi 4m. În laborator au scăpat, dintr-o cutie, 33 de fluturi care s-au împrăştiat, zburând în toată sala, fără a ieşi. Arătaţi că în orice moment există doi fluturi la o distanţă mai mică de 3,50 m unul de altul. Prof.Gh.Efrim, Gazeta Matematică Soluţie: Se împarte paralelipipedul în 32 de cuburi cu latura 2m; un astfel

49 .■ 4 VIII.029 Araţi că dacă în paralelipipedul dreptunghic [ABCDEFG] are

de cub are diagonala 2 3 = 12 =

48 7 < = 4 2

] + Α[ABFE ] + Α[ADHE ] , atunci loc inegalitatea 3 ≤ Α[ABCD 2 2 AC AF 2 AH 2 paralelipipedul este cub. Prof. D.M.Bătineţu-Giurgiu, Bucureşti Soluţie: Notăm AB = CD = EF = GH = x , AD = BC = FG = EH = y şi 38


o. ro

Soluţie: Volumul săpunului este de 64cm3; după 7 zile din săpun au rămas 8cm3; în cele 7 zile s-au consumat 56cm3, deci în fiecare zi s-au consumat 8cm 3, aşadar în a opta zi săpunul dispare. ■ VIII.033 Fie m > n numere naturale. Pentru orice număr natural k

AE = DH = CG = BF = z dimensiunile paralelipipedului; exprimând ariile şi folosind teorema lui Pitagora, inegalitatea propusă devine:

3≤∑

2 xy =S x + y2 2

(1). Cu inegalitatea mediilor avem însă că

notăm a k =

2ab ≤ 1 ⇒ S ≤ 3 (2); din (1) şi (2) obţinem că: S = 3, adică 2 a + b2

⎧x 3 + y 3 + z 3 = x + y + z pentru care avem: ⎨ 2 2 2 ⎩ x + y + z = xyz

k

Soluţie: Ideea e să observaţi : 5 − 2 =

w .n

w 39

)

5 − 2 . Arătaţi că :

Concurs Moldova

Concurs Canada 3 3 Soluţie: Considerăm f ( x, y, z ) = ( x − x) + ( y − y ) + ( z 3 − z ) ;evident prima ecuaţie este echivalentă cu f ( x, y, z ) = 0 . Dacă x,y,z≥1, avem f ( x, y, z ) ≥ 0 cu egalitate dacă şi numai dacă x = y = z = 1; înlocuind aceste valori observăm că a doua egalitate nu e satisfăcută; presupunem, de exemplu, că x < 1 ⇒ x 2 + y 2 + z 2 > y 2 + z 2 ≥ 2 yz > yz > xyz , aşadar sistemul nu are soluţii pozitive. (soluţii echivalente, folosind inegalităţi, au fost primite şi de la elevi; s-a observat că tripletul (0,0,0) verifică egalităţile date). ■ VIII.032 Un săpun are forma unui paralelipiped dreptunghic cu dimensiunile de 2 cm, 4 cm şi 8 cm. În fiecare zi se pierde din săpun aceeaşi cantitate. În câte zile se consumă săpunul ştiind că după 7 zile are din nou forma unui paralelipiped dreptunghic, dar cu dimensiunile exact de două ori mai mici (adică 1, 2, 4 cm)? Concurs Bulgaria

k

a m+ n + a m−n = am ⋅ an .

1

5+2

; în rest, simplă verificare.

Clasa a IX-a 2 2 * x + y − 2 xy IX.024 Arătaţi că : > 2 xy , ∀x, y ∈ ℝ+ , x ≠ y x + y − 2 xy Prof. D.M. Bătineţu-Giurgiu, Bucureşti Soluţie: Se ajunge imediat că inegalitatea e echivalentă cu

( x + y )( x + y − 2 xy ) > 0 , care e adevărată ∀ x , y > 0, x ≠ y , ţinând

cont, de exemplu, de inegalitatea mediilor. ■

eu

de soluţii, de forma: k 30 ⋅ x0 , k 20 ⋅ y 0 , k 15 ⋅ z 0 , k 12 ⋅ t 0 , k ∈ ℤ. ■ VIII.031 Găsiţi, dacă există, tripletele (x , y , z) de numere pozitive

) (

5+2 +

tri n

x = y = z, paralelipipedul fiind astfel cub. ■ VIII.030 a) Scrieţi numărul 3 25 ca o sumă de trei pătrate perfecte; b) Arătaţi că ecuaţia x 2 + y 3 + z 4 = t 5 are o infinitate de soluţii în mulţimea numerelor naturale. Prof.Dr.Dorin Mărghidanu, Corabia Soluţie: a) 3 25 = 3 24 + 3 24 + 3 24 ; b) o soluţie a ecuaţiei date este aşadar: x 0 = 312 , y 0 = 38 , z 0 = 36 , t 0 = 35 şi astfel ecuaţia are o infinitate

(

IX.025

Fie a, b, c ∈ ℕ* , b > a + c şi A={x∈ℝ/ ax 2 + bx + c = 0} .Demonstraţi că : a) A are exact două elemente ; b) A ∩ ℤ are cel mult un element ;

c) Există a, b, c astfel încât A ∩ ℤ are exact un element . Prof.Lucian Dragomir, Oţelu-Roşu Soluţie în RMT 1/2004. ■ IX.026 Determinaţi numărul tripletelor (x , y , z) de numere reale care

satisfac:

⎧ xy = z − x − y ⎪ ⎨ yz = x − y − z ⎪ zx = y − z − x ⎩

Concurs Canada Soluţie: Scădem a doua ecuaţie din prima şi ajungem la (x + 2)(y − z) = 0 ; continuarea nu creează acum probleme (trebuie doar atenţie); se obţin 5 soluţii: (−2,0 − 2), (−2,−2,0), (0,−2,−2), (0,0,0, ), (−1,−1,−1). IX.027 Demonstraţi că dacă numerele strict pozitive a, b, c

40


a b c 3 + + ≥ . b +1 c +1 a +1 2

b şi se deduce a a b b f ( x) ≤ x 2 + ; în a doua inegalitate : x → x − şi în final: b a a

Soluţie: În prima egalitate facem x → x +

Prof.Dorin Mărghidanu, R.M.T. (prelucrare) Soluţie: Putem, de exemplu, să folosim inegalitatea CBS:

a b c ⎞ + + ⎟≥ ⎝ b + 1 c + 1 a + 1⎠

[(a + 1) + (b + 1) + (c + 1)]⋅ ⎛⎜

fracţionară a lui x, [x] este partea întreagă a lui x, iar (x) este cel mai apropriat întreg de numărul real x. Prof. Nicolae Bişboacă, Alba Iulia – Gazeta Matematică Soluţie: Se ajunge imediat la: 9 ⋅ {x} = 2 ⋅ [x ] + ( x ) ∈ ℤ, de unde

⎫ ⎭

4 25 }. ■ 3 9

ajungem la mulţimea soluţiilor: {0, ,

IX.029 O dreaptă variabilă taie laturile AB, BC, CD şi DA ale unui dreptunghi respectiv în M, N, P şi Q. Arătaţi că:

w .n

AB 2 BC 2 + = constant . NQ 2 MP 2

Gazeta Matematică Soluţie: Problemă chiar simplă, trebuie aplicate tot felul de asemănări şi 2

un pic teorema lui Pitagora: ajungem la:

2

AB BC + = 1. ■ 2 NQ MP 2

IX.030 Fie a, b > 0. Determinaţi funcţiile f : ℝ → ℝ pentru care

a b a b b f (x − ) + 2x ≤ x2 + 2 ≤ f (x + ) − 2x , ∀ x ∈ ℝ şi f (0) = . b a b a a

w

Prof.Dr.Dorin Mărghidanu, Corabia

41

unde avem: f ( x ) = x 2 + 1. ■ IX.031 Lui Alin îi place mult cifra 5, dar lui Dragoş nu-i place deloc. Alin a scris toate numerele de 7 cifre care conţin cel puţin o cifră de 5 (i-a cam luat ceva timp!), iar Dragoş a scris toate numerele de 7 cifre în care nu apare cifra 5. Care dintre cei doi a scris mai multe numere (adică a cam pierdut vremea)? *** Soluţie: Determinăm întâi numărul x de numere de 7 cifre cu proprietatea că în niciunul nu apare cifra 5; la fiecare din cele 7 poziţii ale unui astfel de număr putem pune oricare din cifrele 0, 1, 2,…, 9 (fără 5 şi fără 0 pe prima poziţie); avem astfel, cu principiul produsului: x = 8 ⋅ 9 6 = 4251528 . Deoarece avem în total 9000000 numere de 7 cifre avem că numărul de numere de 7 cifre în care avem cel puţin un 5 este 9000000-4251528=4748472, adică Alin a scris mai multe numere decât Dragoş. ■ IX.032 Fie ABC un triunghi cu B şi C fixe, iar A variabil astfel încât tg B + tg C = k (constant). Găsiţi locul geometric al ortocentrului triunghiului ABC. (Gazeta Matematică)

eu

x ∈ ⎨ / k = 0,8⎬ . Se analizează destul de rapid cazurile ce apar şi

a 2 b a x + ; folosim acum ipoteza f (0) = şi deducem a = b, de b a b

tri n

c a b ⎤ ⎡ ≥ ⎢(a + 1) ⋅ + (b + 1) ⋅ + (c + 1) ⋅ =9 a +1 b +1 c + 1⎥⎦ ⎣ x + [x ] + ( x ) IX.028 Rezolvaţi ecuaţia: {x} = , unde {x} este partea 10

⎧k ⎩9

f ( x) =

.■

2

o. ro

au suma egală cu 3, atunci

Soluţie:

BD DC şi la fel: tgB = ⇒ HD HD BC ⇒ HD = const. ⇒ H k = tgB + tgC = HD ctgB1 = tgC =

descrie o paralelă la BC (de fapt, având în vedere că A poate fi în oricare din semiplanele delimitate de BC, H poate fi pe oricare din paralelele la BC situate la distanţa

BC de aceasta). ■ k 42


o. ro

IX.033 Determinaţi cel mai mare număr natural n pentru care

1 1 ajungem la: f ( ) ≥ − ln t sau − f (−t ) ≥ − ln t t t adică f (t ) ≤ ln t , ∀ t > 0. În final: f ( x) = ln x , ∀ x > 0. ■ 1 X.027 Rezolvaţi ecuaţia: log12 x + 4 x = ⋅ log 9 x 2 Dacă x = z = 1 şi y =

n 4 9 este adevărată pentru orice x, y >0. + ≥ inegalitatea x y x+ y

(

Concurs Moldova

⎛4 9⎞ 2 + ⎟⎟( x + y ) ≥ (2 + 3) = 25 , aşadar n ≤ 25. Pentru x = 2, y ⎝x y⎠

Soluţie: ⎜⎜

= 3 valoarea n = 25 verifică inegalitatea dată ⇒ n = 25. ■

ℕ*, x > y.

Prof. D.M. Bătineţu-Giurgiu, Bucureşti Soluţie: Pentru x =2, y =1 se ajunge imediat la f(1) =1; pentru x =3, y =2, folosind injectivitatea lui f ajungem la: f(3 )=3. Presupunem că

orice x ∈ ℝ, atunci a = b = c = d = 0.

π

2

***

, π . Se obţine un

w .n

Soluţie: Dăm valori convenabile lui x, de exemplu 0, sistem chiar foarte simplu de rezolvat. ■

X.026 Determinaţi funcţiile f: ℝ+* → ℝ cu proprietatea:

f ( xyz ) ≥ f ( x) + f ( y ) + f ( z ) ≥

ln (xyz ) ,∀x,y,z∈ℝ+*. 1 + ln x ⋅ ln y ⋅ ln z

Prof. D.M. Bătineţu-Giurgiu, Bucureşti

w

Soluţie: Facem x = t , y = z = 1 şi ajungem la : f (t) ≥ ln t , ∀ t > 0; Punem acum x = y = z = t şi deducem f (1) = 0 . În continuare luăm

1 x = t, y = , z = 1 şi deducem: f (t ) + t 43

t

t

⎛3⎞ ⎛9⎞ ⎜ ⎟ + ⎜ ⎟ = 1 , ecuaţie care are soluţia unică t = 1 ( de ce ? ) ⇒ ⎝ 12 ⎠ ⎝ 12 ⎠

x = 81. ■

X.028 Fie f, g: ℝ→ ℝ două funcţii care satisfac simultan relaţiile: a) f 3 ( x ) + 3 ⋅ f ( x) ⋅ g 2 ( x ) = cos 2 x , ∀ x ∈ ℝ;

b) g 3 ( x) + 3 ⋅ g ( x) ⋅ f 2 ( x) = sin 2 x , ∀ x ∈ ℝ. Arătaţi că: (i) Există x , y ∈ ℝ∖ℚ , x ≠ y , f ( x ) + g ( y ) ∈ ℚ ; (ii) f şi g nu sunt injective. Prof.Dr.Dorin Mărghidanu, Corabia Soluţie: Adunăm şi apoi scădem egalităţile date şi ajungem la:

eu

f(x) = x , ∀ x ≤ n şi se demonstrează că: f(x + 1) = x + 1, dacă facem în enunţ x = n + 1 şi y = n, folosind deasemenea injectivitatea lui f. (Atenţie la redactarea completă, aici aveţi doar idei!). ■ X.025 Arătaţi că dacă a + b cos x + c cos 2 x + d cos 3 x = 0 pentru

)

x +4 x =

tri n

X.024 Determinaţi funcţiile injective f: ℕ*→ℕ* cu proprietatea că f (2) = 2 şi x ⋅ f ( x) + y ⋅ f ( y ) + x ≤ f ( x − y + 1) + 2 xy + y , ∀ x , y ∈

Prof.Manole Neagu – Gazeta Matematică

1 ⋅ log 9 x = t şi 4 x = y ; ajungem 2 imediat la: 12 t = y + y 2 şi 3t = y , de unde 12 t = 3t + 9 t sau

Soluţie: Notăm log12

Clasa a X-a

(

)

⎛1⎞ f ⎜ ⎟ = 0 , ∀ t > 0. ⎝t ⎠

1 1 (1 + 3 cos 2 x ) şi g ( x) = (1 − 3 cos 2 x ) ; e suficient acum să 2 2 luăm, de exemplu, x = 0, y = π şi avem f ( x) + g ( y ) = 1 ∈ℚ; f (0) = f (π ) , deci f nu este injectivă,etc. ■

f ( x) =

X.029 Demonstraţi că dacă ABCD este un patrulater inscriptibil, atunci centrele de greutate ale triunghiurilor ABC, BCD, CDA, ADB sunt conciclice. *** Soluţie:Notăm afixele punctelor A,B,C,D cu a, b, c, respectiv d şi ale centrelor de greutate din enunţ cu x, y, z, respectiv t; deoarece ABCD este

inscriptibil, avem:

c−a d −a : ∈ ℝ. Pe de altă parte: c−b d −b

44


o. ro

a+b+c b+c+d c+d +a a+d +b ; deducem ,y = ,z = ,t = 3 3 3 3 t − y x− y a−c a−d ∈ℝ; concluzia vă aparţine. ■ acum: : = : t − z x− z b−c b−d 1 X.030 Dacă z ∈ ℂ astfel încât z + = 3 , z 1 *** rezultă z n + n ∈ ℝ , ∀ n ∈ ℕ *? z 1 π Soluţie: Putem scrie z + = 3 = 2 ⋅ cos a , cu a = ; se ajunge acum z 6 la ecuaţia z 2 − ( 2 cos a ) z + 1 = 0 care are soluţiile z1, 2 = cos a ± i ⋅ sin a ; nu avem decât de aplicat formula lui Moivre şi

triunghiurile DBC, ECA, FAB au arii egale, atunci triunghiul ABC este echilateral. Prof.Lucian Dragomir, Oţelu-Roşu Soluţie în RMT 1/2004. ■

x=

A = {a n / n ≥ 0}, determinaţi funcţiile injective f: A, A pentru care există

o funcţie surjectivă g: A→ A astfel încât f(g(x)) ≤ g(x), ∀ x ∈ A.

tri n

ajungem la z n +

Clasa a XI-a XI.024 Se dă un şir strict crescător (a n )n≥0 de numere reale. Notând

Prof. D.M. Bătineţu-Giurgiu, Bucureşti Soluţie: Deoarece g este surjectivă avem că există t 0 ∈ At astfel încât

g (t 0 ) = a 0 . Conform enunţului avem: f ( g (t 0 )) ≤ g (t 0 ) ⇔ f (a 0 ) ≤ a 0

de unde deducem: f ( a 0 ) = a 0 . La fel se ajunge la existenţa unui t1 ∈ A

1 ∈ ℝ. ■ zn

X.031 A, B şi C sunt trei puncte pe un cerc de rază R astfel încât

injectivă ⇒ f (a1 ) = a1 . Procedăm acum prin inducţie: presupunem că

f (x) = x , ∀ x ∈ {a 0 , a1 ,..., a n −1 }şi analog se obţine: f (a n ) = a n . Aşadar f : A → A , f (x) = x. ■ XI.025 Fie matricele A, B ∈ Mn ( ℝ ) care satisfac proprietăţile:

w .n

eu

AB = AC ≤ R, iar D este punctul din interiorul cercului astfel încât triunghiul ACD este echilateral; dreapta BD mai intersectează cercul în E. Arătaţi că DE = R . Test tabără naţională Soluţie: Încă aşteptăm o soluţie corectă şi completă (chiar şi de la profesori, nu vă supăraţi pe noi, poate chiar vă aducem o bucurie, ce faceţi în iulie?). ■ X.032 În patrulaterul convex ABCD diagonalele AC şi BD sunt perpendiculare şi se intersectează în E. Arătaţi că simetricele lui E faţă de AB, BC, CD,DA sunt puncte conciclice. Concurs SUA

cu g (t1 ) = a1 şi apoi f (a1 ) ≤ a1 , adică f (a1 ) ∈ {a 0 , a1 } ;cum f este

Soluţie: Considerăm a, bi, ci, di afixele punctelor A, B, C, D (a, b, c, d ∈ ℝ); dacă M, N, P, Q sunt simetricele lui E faţă de AB, BC, CD, DA,

atunci afixele lor sunt: m = abi , n = bci , p = cdi , q = adi ; a + bi c + bi c + di a + di

p−m q−m ∈ℝ⇒M,N,P,Q conciclice.■ : p−n q−n

w

Calcule imediate conduc la:

X.033 Notăm cu D, E, F mijloacele arcelor mici BC, CA, respectiv AB ale cercului circumscris triunghiului ascuţitunghic ABC. Arătaţi că dacă

45

a) Există m ∈ ℕ , m ≥ 2 , astfel încât Am = Am+1; b) A + B = I n . Arătaţi că matricea I n – AB este inversabilă. Prof. D.M. Bătineţu-Giurgiu, Bucureşti Soluţie: B = I n − A ⇒ AB = A( I n − A) = A − A 2 şi la fel

BA = A − A 2 , deci AB = BA ; conform a) avem : A m ( I n − A) = On sau A m ⋅ B = On . Se obţine astfel:

( AB) m = A m ⋅ B m = A m ⋅ B ⋅ B m −1 = On şi deci:

I n = I n − On = I n − ( AB ) m = ( I n − AB)( I n + AB + ... + ( AB) m −1 ) ⇒ I n – AB este inversabilă. ■ XI.026 Arătaţi că dacă A ∈ M n ( ℂ ), atunci există o infinitate de numere complexe α astfel încât matricele α ⋅ I n + P( A) să fie inversabile, oricare ar fi polinomul P cu coeficienţi întregi.

46


o. ro

Prof.Univ.Dr.Constantin Niculescu, Gazeta Matematică Soluţie: Încă n-am primit ceva, vă mai aşteptăm (vara nu-i ca iarna).■ XI.027 Dacă 0 < a < b şi x n = b + ( −1) n −1 a , n ∈ ℕ *, calculaţi

x lim n +1 şi lim n x n . x →∞ x n →∞ n

a< b cu f (a) = f (b) am avea că f este constantă pe [a, b], deci ∀c ∈ (a, b)

avem f (c – 0) = f (b – 0), contradicţie cu (b) ⇒f (a) < f (b), adică f este chiar strict crescătoare. ■

, ∀ m , n ≥ 1; b) lim x n 2 = 1 . n →∞

Demonstraţi că: lim x n = 1 .

XI.032 Fie f: ℝ→ ℝ o funcţie cu proprietatea lui Darboux astfel încât f D f este injectivă. Arătaţi că f D f este strict crescătoare. Gazeta Matematică Soluţie: mai aşteptăm (nefiind publicată, soluţia corectă se va puncta). ■

w .n

Conf.Univ.Dr.Cristinel Mortici, Gazeta Matematică Soluţie: mai încercaţi, este timp. ■

tg (ax) − sin( ax) , unde a ,b ∈ ℝ*, fără a folosi x →0 tg (bx ) − sin(bx )

***

⎡ π⎤ Soluţie: Se consideră funcţia derivabilă f : ⎢0, ⎥ → ℝ şi studiind ⎣ 2⎦

variaţia ei se ajunge că x =

π

4

este punct de minim, deci

2n

⎛ 2⎞ ⎟ este valoarea minimă a lui f; deducem că f ( ) = 2 ⋅ ⎜⎜ ⎟ 2 4 ⎝ ⎠ Im f = (0,1] ;concluzia e imediată. ■

π

n →∞

⎡ π⎤ ⎥ astfel încât ⎣ 2⎦

sin 2 n x + cos 2 n x = a

eu

Prof.Univ.Dr.Ion Chiţescu, Gazeta Matematică Soluţie: La fel, mai aveţi o şansă. ■ XI.029 Un şir ( x n )n≥1 are proprietăţile:

XI.030 Calculaţi lim

enunţ ⇒ f (a) ≤ f (b), adică f este crescătoare pe ℝ. Dacă există a, b ∈ℝ ,

XI.033 Arătaţi că ∀ a ∈ (0, 1] , ∃ n ∈ ℕ şi ∃ x ∈ ⎢0,

b) A 2 = O3 .

m+n

Avem acum f (c – 0) ≥f (b) ≥ f b – 0), contradicţie cu condiţia (b) din

tri n

că pentru orice n ∈ ℕ avem: A n = x n ⋅ I 3 + y n ⋅ A ;

m−n

deducem că există c ∈ (a , b) astfel încât f (x) >f (b) , ∀ x ∈ (a , c).

Gazeta Matematică

Soluţie: Aşteptăm (oricum, dacă nu reuşiţi şi gata, vom publica).■ ⎛ c 1 − 1⎞ XI.028 Fie a, b, c ∈ ℂ cu a ≤ 1, b ≤ 1, c ≤ 1 şi A = ⎜⎜ 0 b 0 ⎟⎟ . ⎜1 1 a ⎟ ⎝ ⎠ Arătaţi că următoarele afirmaţii sunt echivalente: a) Există două şiruri ( x n ), ( y n ) de numere complexe cu proprietatea

a) x m − x n ≤

Soluţie: Pentru a < b, presupunând f(a) >f(b) , din f(a + 0) ≥f(a)>f(b)

Clasa a XII-a

b) pentru orice a, b ∈ ℝ , a < b, avem : f ( a – 0 ) < f ( b – 0 ). Demonstraţi că f este strict crescătoare . Prof.M. Piticari, S. Rădulescu- Gazeta Matematică

XII.024 Determinaţi funcţiile f : (ℤ p , +)→ (ℤ p , +), cu p număr prim, care satisfac: f ( x + f ( y )) = y + f ( x) , ∀ x , y ∈ ℤ p . Conf.Univ.Dr.Cristinel Mortici, R.M.T. Soluţie: mai încercaţi. ■ XII.025 Fie (M, *) un monoid şi e elementul său neutru. Este posibil să existe H ⊂ M aşa încât (H, *) să fie grup, iar elementul neutru al lui H să fie diferit de e? Prof. Liliana Niculescu, Craiova – Gazeta Matematică Soluţie: aşteptăm, poate vara aveţi mai mult timp liber. ■

47

48

derivatele.

Gazeta Matematică

3

⎛a⎞ Soluţie: ⎜ ⎟ . ■ ⎝b⎠

XI.031 Se consideră f : ℝ → ℝ o funcţie care satisface condiţiile:

f(a – 0 ) ≤ f ( a ) ≤ f ( a + 0);

w

a) f are limită în orice punct a ∈ ℝ şi


o. ro

XII.026 Fie ( A,+,⋅) un inel şi a, b ∈ A cu proprietăţile:

π

2

x

∫ 1 + sin x + cos x dx

(i) Există m ∈ ℕ, m ≥ 2 astfel încât am = am+1; (ii) a + b = 1. Arătaţi că 1 – ab este inversabil. Prof. D.M.Bătineţu-Giurgiu, Bucureşti Soluţie: Urmăriţi soluţia problemei XI.025 . ■

XII.031 Calculaţi integrala :

XII.027 Fie f : ℝ → ℝ o funcţie derivabilă cu proprietăţile: a) Există lim x ⋅ f '( x ) = L;

XII.032 Fie (G,⋅) un grup multiplicativ şi H un subgrup al său astfel încât dacă M este subgrup al lui G şi M este izomorf cu H, atunci M = H. Arătaţi că ∀ g ∈ G şi ∀ h ∈ H ⇒ ghg −1 ∈ H. Prof.Florin Nicoară, Oradea Soluţie: Fie g ∈ ∀; încercăm să arătăm că gHg −1 = H unde

0

Prof. Drd.Manuela Prajea, Drobeta Tr.Severin

Soluţie: Schimbarea de variabilă: x =

b) Dacă F este o primitivă a lui f , atunci lim

x →∞

F ( x) =1 x

Calculaţi L şi lim f ( x ) . x →∞

Prof.Mihai Bălună, Gazeta Matematică Soluţie: mai aşteptăm. ■ ln 2

∫ arctg (e

XII.028 Calculaţi integrala :

x

− 1)dx .

0

8

Prof.D.M. Bătineţu-Giurgiu, Bucureşti

⋅ ln 2 . ■ 1

XII.029 Calculaţi integrala:

x2 ∫ x dx −1 1 + e

w .n

Prof.Lucian Tuţescu, Craiova Soluţie: Facem, de exemplu, schimbarea de variabilă x = −t şi integrala −1

1

t2 et ⋅ t 2 ( 1 ) − dt = dată va fi egală cu I = ∫ −t ∫ t dt = J calculăm acum 1 1+ e −1 1 + e 2 1 I + J = = 2I ⇒ I = . ■ 3 3 XII.030 Fie (G ,⋅) un grup multiplicativ comutativ şi a ∈ G. Dacă funcţia f : G → G are proprietatea că f ( x) ⋅ ( f D f )( x) = a ,∀x∈G,

w

4

⋅ ln 2 .■

XII.033 Pentru ce valoare a lui m > 0 aria mulţimii

6⎫ ⎧ A = ⎨( x, y ) / m ≤ x ≤ 2m,0 ≤ y < x + 2 ⎬ este minimă ? x ⎭ ⎩ 2m 6 Soluţie: Aria mulţimii este egală cu ∫ ( x + 2 ) dx = g ( m) ; nu avem x m

decât să studiem variaţia funcţiei g, pentru m > 0; obţinem m = 1 punct de minim. ■

arătaţi că f este injectivă dacă şi numai dacă este surjectivă . Prof.D.M. Bătineţu-Giurgiu, Bucureşti Soluţie: mai aşteptăm.■

49

π

gHg −1 = {gxg −1 / x ∈ H } . Deoarece ( gag −1 )( gbg −1 ) = gag −1 gbg −1 = g (ab) g −1 şi ab ∈ H, deducem că gHg −1 este un subgrup al lui G. E suficient acum să observăm că funcţia f : H → gHg −1 , f ( x) = ghg −1 este un izomorfism. ■

eu

Soluţie:

π

2

− t ; obţinem în final:

tri n

x →∞

π

50


o. ro

(Informaţii suplimentare se pot obţine la: prof. Lucian Dragomir, tel: 0255/530303 sau 0722/883537). ■

Concursul Judeţean al Revistei de Matematică Caraş-Severin , Ediţia a II-a

Notă: Rugăm toţi colaboratorii care ne trimit probleme propuse obligatoriu cu soluţii!), note, articole, etc., să tehnoredacteze materialele pe calculator (daca e posibil) şi să le ataşeze ca fişier la mesajul lor, apoi să folosească adresa: lucidrag@yahoo.com (cu menţiunea: materiale pentru RMCS).

tri n

Probleme propuse

( pentru participare la concurs şi nu numai )

(Data limită de trimitere a soluţiilor: 30 iulie 2006) Clasa a IV-a

IV.011 Câtul împărţirii a două numere este 7, iar restul 13. Aflaţi cele două numere ştiind că diferenţa lor este 613. Înv. Marioara Popescu, Reşiţa

w

w .n

eu

Regulament Ediţia a II a a Concursului Revistei este în plină desfăşurare, urmează un nou set de probleme. Fiecare elev trebuie să rezolve (subliniem din nou: singur!; altfel e posibil să vă treziţi calificaţi la concurs şi acolo să nu faceţi mare lucru→ daţi naştere la întrebări şi credem că nici n-o să vă simţiţi prea bine), aşadar să rezolve cât mai multe probleme de la clasa sa, de la clasa precedentă sau de la orice clasă superioară (am avut anul acesta multe situaţii şi de acest gen). Redactaţi îngrijit fiecare problemă pe câte o foaie separată (enunţ + autor + soluţie + numele vostru), completaţi talonul de concurs de pe ultima pagină a revistei şi trimiteţi totul într-un plic (încercaţi să fie unul ceva mai mare, format A5 cel puţin -nu înghesuiţi tot în ceva mic) adresat astfel: Prof.Lucian Dragomir , Grup Şcolar Industrial Oţelu-Roşu, str.Republicii 10-12, 325700, Oţelu-Roşu, Caraş-Severin, cu menţiunea “probleme rezolvate”. Insistăm asupra trimiterilor în plic (nu în folii de plastic) şi asupra respectării cu stricteţe a termenelor finale indicate de fiecare dată - plicurile primite după data limită nu vor fi luate în considerare. După data limită de trimitere a soluţiilor, acestea sunt evaluate şi în numarul următor al revistei vor fi publicaţi toţi rezolvatorii cu punctajele obţinute. La ediţia a II-a a concursului vor fi selectaţi concurenţii în funcţie de punctajele obţinute din rezolvarea problemelor publicate în numerele 15, 16, 17 şi 18 ale revistei noastre. În jurul datei de 20 ianuarie 2007 se va întocmi clasamentul general (prin însumarea punctelor obţinute) şi astfel primii clasaţi vor fi invitaţi, împreună, ca şi în acest an, să participe la concurs; acesta va avea loc tot la începutul lunii februarie într-un oraş care va fi anunţat în timp util. Subiectele vor fi alese tot din probleme de genul RMCS sau G.M. sau ceva cât de cât nou. Noutatea ediţiei din acest an constă în faptul că ne adresăm de-acum şi elevilor de ciclu primar. Am demarat cu această ocazie şi un concurs (cu premii din nou) de probleme propuse de către elevi; acestea trebuie trimise în plic separat de eventualele probleme rezolvate, cu menţiunea “ Probleme Propuse“. Încercaţi ! Oricum , n-o să vă pară rău ( am si primit deja ceva ). Spor la treabă tuturor: elevi, profesori, părinţi sau prieteni!

51

IV.012 În vacanţă, Marius a citit luni, marţi şi miercuri o carte (a terminat-o!). Miercuri a citit cu 100 de pagini mai mult decât luni şi marţi la un loc.Marius constată că miercuri a citit de trei ori mai mult şi încă 4 pagini decât în primele două zile. Câte pagini a citit în fiecare zi şi câte pagini are cartea dacă marţi a citit de două ori mai multe decât luni ? Înv. Marioara Popescu, Reşiţa

IV.013 Daniel este întrebat de tatăl său: “Câţi elevi sunteţi în clasă? “. Daniel răspunde: “Dacă ar mai fi încă o dată pe câţi elevi sunt şi încă pe jumătate şi încă un sfert, împreună cu învăţătorul ar fi 100 de persoane!“ Câţi colegi are Daniel? Inst. Ozana Sărin, Reşiţa IV.014 Câte caiete se pot cumpăra în loc de un dicţionar, ştiind că un dicţionar costă cât 10 cărţi, 5 cărţi cât un penar şi două penare cât 5 stilouri, iar 10 stilouri cât 100 de caiete? Înv. Ana Modoran, Reşiţa

52


o. ro

IV.015 Într-o clasă sunt 30 de elevi. Câţi băieţi şi câte fete sunt în clasă ştiind că dacă ar fi cu doi băieţi mai puţin, atunci jumătate din numărul lor ar reprezenta de două ori mai mult decât o treime din numărul fetelor ? Înv. Ana Modoran, Reşiţa

IV.017 Dacă mărim două din laturile nealăturate ale unui pătrat cu câte 6 cm, figura obţinută va avea perimetrul de 44 cm. Aflaţi aria figurii iniţiale. Inst. Lidia Todor, Caransebeş

V.034. Fie a = 726 + 728 ⋅ 726 + 729 ⋅ 3 Să se stabilească valoarea de adevăr a propoziţiei: „ a este pătrat perfect şi cub perfect” Prof. Mariana Drăghici, Reşiţa 228 171 V.035 Care dintre numerele 124 şi 626 este mai mare? Prof. Mariana Drăghici , Reşiţa V.036 Să se determine cifrele a, b, astfel încât numărul A = aabaa , scris în baza 10 să fie divizibil cu 109. Prof. Loreta Ciulu, Reşiţa

w .n

eu

IV.018 La un magazin de ceasuri, noul model are succes: în prima zi s-au vândut 2/5 din total şi încă 5 ceasuri. A doua zi s-au vândut 1/4 din rest şi încă 3 ceasuri. A treia zi s-au mai vândut 2/3 din câte rămăseseră şi încă 2 ceasuri. În raft mai sunt 4 ceasuri. Câte ceasuri au fost aduse spre vânzare? Inst. Lidia Todor, Caransebeş IV.019 Trei persoane au mers cu o maşină pentru care au plătit 350 000 lei; prima persoană a mers 15 km, a doua 25 km şi a treia 30 km. Cât a plătit fiecare pentru drumul parcurs? Înv. Mirela Tătar, Caransebeş

tri n

IV.016 Mihai are 6 ani. Peste 6 ani, vârsta sa va fi exact de trei ori mai mică decât a mamei sale, iar a bunicului de două ori mai mare decat a fiicei sale. Ce vârstă are mama? Dar bunicul? Petra-Ana Rogge, elevă, Reşiţa

IV.022 Din Caransebeş porneşte spre Constanţa un automobil cu viteza de 45 km/oră. După trei ore, tot din Caransebeş porneşte spre Constanţa un alt automobil care se deplasează cu 60 km/oră. Aflaţi: a)după câte ore al doilea automobil îl ajunge pe primul; b)la ce distanţă de Caransebeş se întâlnesc cele două automobile. Înv. Mirela Tătar, Caransebeş IV.023 Dublând suma a două numere naturale am obţinut 150. Aflaţi diferenţa numerelor ştiind că primul este de patru ori mai mare decât al doilea. Inst. Adriana Ursu, Caransebeş IV.024 Un sac cu ciment cântăreşte de două ori mai mult decât un sac cu var. Dacă 7 saci cu var şi 7 saci cu ciment cântăresc împreună 525 kg, cât cântăreşte un sac cu ciment ? Inst.Mariana Ionescu, Caransebeş Clasa a V-a

IV.020 Ioana propune prietenilor ei o problemă: Andrei este fratele meu; într-o zi el îmi spune: “Am tot atâţia fraţi câte surori am.“. “E adevărat, îi răspund eu, dare u am de două ori mai mulţi fraţi decât surori.“. Câţi băieţi şi câte fete sunt în familia celor doi? ***

w

IV.021 Jerry se găseşte la 20 de paşi de adăpostul său. Tom se găseşte la 5 paşi de şoricel. Când pisica face o săritură, Jerry face 3 paşi, iar o săritură a pisicii este cât 10 paşi de şoricel. Poate Tom să-l prindă pe Jerry? Concurs Bulgaria

53

V.037 Se dă numărul A = 4 n +1 ⋅ 5 2 n + 3 − 2 , n ∈ ℕ a) Găsiţi de câte ori apare cifra 9 în scrierea zecimală a lui A; b) Arătaţi că numărul care reprezintă suma cifrelor lui A este divizibil cu 3. Prof. Zoran Ocanovici, Moldova-Nouă V.038 Se înşiră unul după altul toate numerele naturale de la 1 până la 2006 fără a lăsa spaţiu între ele şi fără a pune virgulă. Se obţine numărul n = 1234567891011...20052006. a) Determinaţi câte cifre are acest număr; b) Aflaţi suma primelor 500 de cifre ale lui n. Prof. Mariana Iancu, Oraviţa V.039 Determinaţi câte numere de 11 cifre încep şi se termină cu 2006. Câte dintre acestea au suma cifrelor egală cu 22 ? Prof. Mariana Iancu, Oraviţa

54


o. ro

Clasa a VI-a VI.034 Să se determine trei numere prime având suma egală cu 44 , ştiind că unul dintre ele este de forma aa . Prof. Emilia-Dana Schiha , Berzasca VI.035 Să se calculeze suma tuturor numerelor de forma ababab ştiind

tri n

că fiecare astfel de număr are exact 24 de divizori şi ab este număr prim. Prof. Marius Şandru , Reşiţa VI.036 Bisectoarele a două unghiuri adiacente formează un unghi cu măsura de 600.Aflaţi măsura fiecărui unghi ştiind că măsura unuia este de patru ori mai mare decât a celuilalt. Prof. Vasile Huza , Coronini VI.037 Găsiţi numerele naturale x şi y , x≤y , care satisfac : 3x + 3y = 738 Prof. Vasile Huza , Coronini VI.038 În triunghiul ABC m(∠B ) + m(∠C ) reprezintă 0,8 din

m(∠A) ,iar m(∠B ) − m(∠C ) = 10 0 .Aflaţi:

a ) Măsurile unghiurilor triunghiului ABC; b ) Măsura unghiului format de bisectoarea lui ∠BAC şi înălţimea din B . Prof. Ion Belci , Reşiţa VI.039 După o mărire de 2 %, apoi alta de 3 % , urmată de o reducere a preţului cu 4 % , o imprimantă costă 504,288 lei ( noi ). Cât a costat iniţial imprimanta ? Prof. Emilia – Dana Schiha , Berzasca VI.040 Câte triunghiuri cu laturile de lungimi numere naturale există ştiind că numai o latură a unui astfel de triunghi are lungimea strict mai mare decât 3 ? Prof. Romanţa şi Ioan Ghiţă , Blaj VI.041 17 puncte situate în plan determină 46 de drepte. Aflaţi câte dintre punctele date sunt coliniare. Prof. Liviu Smarandache , Craiova VI.042 Arătaţi că numărul A = 200...06 nu poate fi pătrat perfect (indiferent câte cifre de 0 avem ) . Prof. Adriana Dragomir , Oţelu-Roşu VI.043 În pătratul unui număr a , de mai multe cifre , pe locul zecilor se află cifra 7. Ce cifră se află pe locul unităţilor în numărul a 2 ? Concurs Ungaria

eu

V.040 Găsiţi cel mai mare număr natural de forma abab care are cel mai mic număr de divizori. Prof. Vasile Huza, Coronini 2 3 2006 V.041 Fie N = 2 + 2 + 2 + ... + 2 . Ştiind că x este suma dintre restul împărţirii lui N – 2006 la 10 şi un număr prim, iar k este un număr natural par astfel încât 2006 = k ⋅ x , determinaţi valorile posibile ale lui x . Prof. Georgeta Bihoi, Reşiţa V.042 a) Este numărul a = 1 ⋅ 2 ⋅ 3 ⋅ ... ⋅ 2006 + 2007 pătrat perfect? b) Este numărul b = (1 + 2 + 3 + ... + 2000) ⋅ 20012000 + 2006 cub perfect ? Prof. George Pascariu, Bozovici V.043 Arătaţi că n = 1 + 3 + 5 + ... + 2005 este pătrat perfect. Prof. Nicolae Dragomir, Tudor Deaconu, Reşiţa V.044 O foaie de hârtie este ruptă în 3 bucăţi; una dintre acestea se rupe deasemenea în 3 bucăţi, apoi o bucată din cele avute se rupe din nou în 3 bucăţi. Continuând procedeul, se poate obţine un total de 2006 de bucăţi? Dacă da, după câte operaţii? Dar 2007 de bucăţi – se pot obţine? Prof. Heidi Feil, Oţelu-Roşu V.045 Se consideră numărul A = abbc .

w .n

a) Cifra A se înlocuieşte cu cifra d şi se obţine numărul B = dbbe , astfel încât e = c − d .Determinaţi câte numere A există pentru care A − B = 2006 ; b) Schimbând ordinea cifrelor lui A obţinem un număr C. Arătaţi că A − C ≠ 2006 . Prof.Lucian Dragomir, Oţelu-Roşu V.046 Găsiţi pătratele perfecte de forma aabb . Prof.Romanţa şi Ioan Ghiţă, Blaj V.047 Dacă a < b sunt numere naturale consecutive, arătaţi că:

b n − ab n −1 − ab n − 2 − ... − ab 2 − ab = b ∈ ℕ , ∀ n ∈ ℕ , n ≥ 2.

w

Prof. Afilon Moica, Zăvoi V.048 Determinaţi numerele naturale nenule care împărţite la 5 dau câtul a şi restul b şi împărţite la 9 dau câtul b şi restul a. Câte soluţii are problema ? Prof. Adriana Dragomir, Oţelu-Roşu

55

56


o. ro

VI.044 În anul 1066 la Hastings s-a dat o mare bătălie între oştile saxone şi cele normande. Pe ţărm , saxonii au format un pătrat, iar pe plajă normanzii au format alt pătrat . Se spune că normanzii întreceau pe saxoni cu 500 de pedestraşi şi 12 călăreţi. Atacând cu mult curaj , saxonii au ucis jumătate din cotropitori , ei nepierzând decât o cincime din ostaşi.La sfârşitul bătăliei cele două tabere aveau exact acelaşi număr de oameni. Câţi saxoni şi câţi normanzi au luat parte la luptă ? *** VI.045 Pe un cerc se aşează la întâmplare 5 numere naturale a căror sumă este egală cu 18. Arătaţi că există cel puţin două numere alăturate a şi b

a ( a , b ∈ ℕ* ) cu b

2 7− n

BC

w

[AM ] ≡ [MN ] ≡ [ND ] . Să se determine aria triunghiului

este paralelă cu AM şi EF =

AB

3

1 AM . 2

Prof. Nicolae Stăniloiu , Bocşa VII. 037 Rezolvaţi în ℤ ecuaţia : xy + 3 y − 5 x − 17 = 0 . Prof. Emilia-Dana Schiha , Berzasca VII. 038 Demonstraţi că dacă a , b , c sunt lungimile laturilor unui

triunghi , atunci :

a b c b c a + + − − − < 1. b c a a b c

Prof. Nicolae Dragomir,Tudor Deaconu,Reşiţa VII.039 Câte numere de forma abcde sunt divizibile cu 11 şi satisfac a+e=b+d. Prof. Nicolae Stăniloiu , Bocşa VII. 040 Determinaţi numerele reale x ≥

x−

2 pentru care : 3

x 2 2 − = . 3 9 3 Prof. Ion Belci , Reşiţa

VII. 041 Determinaţi x ∈ ℕ* şi y prim pentru care

1 y + = 3. x x2

Prof. Dumitru Bătineţu-Giurgiu , Bucureşti VII.042 Determinaţi x care satisface :

1 + 2 0 + 21 + ... + 2 499 = 2 x .

Prof. Lenuţa Andrei , Craiova VII.043 Arătaţi că ecuaţia x 2 + y 2 + z 2 = 2007 nu are soluţii în ℤ. Prof. Adriana Dragomir , Oţelu-Roşu

ABC ştiind că NC=14 cm.

Prof. Mariana Drăghici , Reşiţa

57

CA

Dacă E este mijlocul lui[NP] şi F mijlocul lui [BC] , demonstraţi că EF

w .n

proprietatea că , adunând şi la numitor şi la numărător acelaşi număr x > 0 , obţinem o putere cu exponent natural a fracţiei iniţiale . Prof. Marius Damian , Brăila VI.048 Fiecare element al mulţimii A = { 1 , 2 , 3 , … , 100 } se colorează cu una dintre culorile roşu , galben sau albastru , respectând următoarele reguli; a ) suma dintre orice număr galben şi orice număr albastru este divizibilă cu 3 ; b ) suma oricăror două numere roşii este divizibilă cu 3. 1 ) Arătaţi că numărul 3 este roşu ; 2) Calculaţi suma tuturor numerelor care nu sunt roşii . Prof.Marius Damian , Brăila Clasa a VII-a VII.034 În triunghiul ABC bisectoarele interioare ale unghiurilor B şi C intersectează mediana [AD ] în M şi N astfel încât

este număr întreg?

Prof. Loreta Ciulu , Reşiţa VII. 036 Se consideră un triunghi ABC şi punctele M , N , P pe laturile [BC] , [AC] , respectiv [AB] astfel încât : BM = CN = AP = 1

eu

VI.047 Determinaţi toate fracţiile de forma

7 +2 n

tri n

*** astfel încât a + b ≥ 8. VI.046 Considerăm segmentul [AB] de lungime 1 m . În interiorul segmentului desenăm cu culoare roşie punctele C 1 , C 2 , … , C 24 , care împart [AB] în segmente congruente de lungime 6 cm. Colorăm apoi cu albastru punctele D 1 , D 2 , … , D 19 care împart [AB] în segmente congruente de lungime 5 cm. Precizaţi câte puincte albastre se suprapun peste puncte roşii şi câte segmente de lungime 1 cm apar pe desen. Prof. Mihai Monea , Deva

VII. 035 Pentru câte valori naturale ale lui n, expresia

58


o. ro

VII.044 Determinaţi restul împărţirii la 7 a numărului A = 76 2007 + 78 2006 . Prof. Heidi Feil , Oţelu-Roşu VII.045 O mulţime A având 4 elemente numere raţionale se numeşte

⎧ 3 x1 − 5 x 2 + 2 x3 ≥ 0 ⎪ 3x − 5x + 2 x ≥ 0 3 4 ⎪ 2 ⎪⎪ ............................. ⎨ ⎪3 x n −1 − 5 x n + 2 x 2 ≥ 0 ⎪ 3 x n − 5 x1 + 2 x 2 ≥ 0 ⎪ ⎪⎩ x1 + x 2 + ... + x n = na

x ∈ A . Notăm 2

Sn = { 1 , 2 , 3 , ... , n } , n ∈ ℕ* . a ) Determinaţi câte submulţimi interesante are S10 ; b ) Determinaţi cel mai mic număr natural n pentru care S2n are cel puţin 2006 de submulţimi interesante. Prof.Lucian Dragomir , Oţelu-Roşu VII.046 Fie ABC un triunghi , I centrul cercului său înscris şi AI ∩ BC = {D} . O dreaptă perpendiculară pe AI intersectează (AB) şi (AC) în punctele P şi respectiv Q;notăm cu M şi N simetricele lui P şi Q faţă de dreptele BI şi respectiv CI. Demonstraţi că MD = DN dacă şi numai dacă AB = AC.

x − y + 3 + 3 y − 2x + 4 + x − 2 y + 5 = 6

b ) Rezolvaţi ecuaţia :

w .n

Prof. Petrişor Neagoe , Anina VII.048 Demonstraţi că orice triunghi cu aria mai mare decât 1 nu poate avea perimetrul mai mic decât 4 . Prof.Lucian Dragomir , Oţelu-Roşu Clasa a VIII-a VIII.034 Să sedetermine numerele naturale pătrate perfecte de forma 4n 2 -15, n ∈ N. Prof. Mariana Drăghici , Reşiţa VIII.035 Fie numerele reale x, y cu proprietăţile: x − 6 y + 2 = 0 şi

w

x ∈ [− 2;4] .Să se determine numărul

a = x + y + 4 x + 4 + x + y − 8 x − 2 y + 17 . 2

Prof. Nicolae Stăniloiu , Bocşa VIII.037 Determinaţi câte perechi ( x , y ) de numere întregi satisfac:

x 2 + y 2 = 3x − y

Prof. Adriana şi Lucian Dragomir , Oţelu-Roşu VIII.038 Fie ABCD un dreptunghi .Arătaţi că nu există un punct M în spaţiu , nesituat în planul dreptunghiului dat ,astfel încât : MD ⊥ ( ABCD) , m(∠MBC ) = 60 0 şi m(∠MBA) = 30 0 . Prof. Ion Belci , Reşiţa

eu

Prof.Marius Damian , Brăila VII.047 . a ) Arătaţi că dacă a, b , c ∈ ℝ şi a + b + c = 6 , atunci a 2 + b 2 + c 2 ≥ 12

, a ∈ ℝ.

tri n

interesantă dacă : ∀ x ∈ A ⇒ 2x ∈ A sau

VIII.036 Determinaţi numerele reale x1 , x 2 ,..., x n care satisfac:

2

2

59

2

VIII.039 Determinaţi funcţiile f : ℝ → ℝ care satisfac :

f ( x – 2 ) ≤ 7 – 2x şi f ( 2 – x ) ≥ 2x – 1 , ∀ x ∈ ℝ . Prof. Mirela Rădoi , Reşiţa VIII. 040 Fie ABC un triunghi dreptunghic în A şi α un plan care trece prin mijlocul M al ipotenuzei şi este perpendicular pe mediana AM. Demonstraţi că există β ∈ ℕ astfel încât pentru orice punct P ∈ α să avem : β ⋅ PA 2 = PB 2 + PC 2 .

Prof. Vasile Huza , Coronini VIII.041 Un paralelipiped dreptunghic cu dimensiunile a, b, c ∈ (0,1] are aria totală 2s . Demonstraţi că :

a+x b+x c+x + + ≥ 3 + s ⋅ x , ∀ x ∈ ℝ +. b c a Prof. Dumitru Bătineţu-Giurgiu , Bucureşti VIII.042 Fie a , b , c ∈ ℝ astfel încât :

a 2 + b 2 + c 2 + ab + bc = 1 .Arătaţi că : b ≤ 2 . Prof. Mirela Genoiu , Craiova

Prof. Loreta Ciulu , Reşiţa

60


o. ro

VIII.043 Demonstraţi inegalitatea :

IX.035 Demonstraţi că :

1 x y z + + ≥ x + 2 y + 3z y + 2 z + 3x z + 2 x + 3 y 2

3+

z − y x − z y − x x2 y2 z2 + + ≤ 2 + 2 + 2 . x y z y z x

m , n ∈ ℕ astfel încât

x + y = m şi

1 1 + =n . x y

IX.037 Determinaţi funcţiile f : ℝ → ℝ care verifică :

w .n

Prof. Nicolae Dragomir , Prof. Tudor Deaconu , Reşiţa VIII.048 Determinaţi numerele naturale nenule n pentru care există numerele naturale x1 , x 2 ,..., x n astfel încât

1 1 1 1 + + ... + = 1+ . x1 x 2 xn 4n

x1 + x 2 + ... + x n = 3n şi

Prof. Lucian Dragomir , Oţelu-Roşu

Clasa a IX-a

IX. 034 Fie ABC un triunghi şi AD , BE , CF trei ceviene concurente în

PA 2 PB 2 PC 2 + + = 12 . PD 2 PE 2 PF 2

w

P. Determinaţi poziţia punctului P dacă

f ( x 4 + y 3 + z 2 + t ) = f ( x) + f ( y 2 ) + f ( z 3 ) + f (t 4 ) , ∀x,y,z,t∈ ℝ . Anca Tuţescu , elevă , Craiova

IX.038 Rezolvaţi ecuaţia :

⎡ x + 1⎤ ⎡ x + 2 ⎤ ⎡ x + 3 ⎤ ⎡ x + 1⎤ x − 1 ⎢⎣ 4 ⎥⎦ + ⎢⎣ 4 ⎥ + ⎢ 4 ⎥ = ⎢ 2 ⎥ + 2 . ⎦ ⎣ ⎦ ⎣ ⎦

Dragoş Unguraş , elev , Oţelu-Roşu IX.039 Determinaţi câte elemente are mulţimea { x ∈ ℝ / x 2 ⋅ ( x + 1) ⋅ ( x + 3) = 2 x + 1 } . Prof. Lucian Dragomir , Oţelu-Roşu IX.040 Demonstraţi că există o ecuaţie de gradul al doilea cu coeficienţi întregi care are o rădăcină egală cu ctg 22,5 0 . Concurs Ungaria IX.041 Demonstraţi că o progresie aritmetică infinită şi neconstantă nu poate avea toţi termenii numere prime. Concurs Ungaria IX.042 În paralelogramul ABCD o paralelă la AC intersectează drepteleAB si AC în punctele M şi N (M∈AB, N∈BC). Paralela d 1 prin

eu

Prof. Marius Damian , Brăila VIII.046 Numărul x are 4n cifre , toate egale cu 3 , numărul y are 2n cifre , toate egale cu 9 . Determinaţi cel mai mic număr natural z pentru care numărul A = 3 x + 2 y + z este pătrat perfect pentru orice n ∈ ℕ* . Prof.Marian Bădoi , Oraviţa VIII.047 . Determinaţi numerele raţionale x şi y pentru care există

Prof. Dumitru Bătineţu-Giurgiu , Bucureşti IX.036 Demonstraţi că un patrulater ABCD convex şi ortodiagonal este romb dacă şi numai dacă AC 2 + BD 2 = AB 2 + CD 2 + AB ⋅ CD + BC ⋅ AD . Prof. Daniel Jinga , Piteşti

tri n

Prof. Liviu Smarandache , Craiova VIII.044 O clădire are 72 de etaje şi este dotată cu un lift special. Dacă se apasă butonul galben liftul urcă 7 etaje , iar dacă se apasă butonul verde liftul coboară 9 etaje ( dacă una din aceste comenzi nu e realizabilă liftul rămâne pe loc ) . Putem ajunge cu acest lift de la etajul 1 la etajul 72 ? Concurs Bulgaria VIII.045 a ) Arătaţi că pentru orice x , y > 0 avem : x 3 + y 3 ≥ x 2 y + xy 2 ; b ) Dacă x , y , z > 0 , demonstraţi că :

2 ⋅ ( x + y − xy ) ≥ x 2 + y 2 , ∀ x , y ∈ ℝ+*

M la BC şi paralela d 2 prin N la AB se intersectează in E. Notăm cu P mijlocul segmentului [MN], cu T mijlocul lui [RS], unde {R}= d1 ∩ AC,{S} = d2 ∩ AC.Demonstraţi că B,D,E,P,T sunt coliniare. Prof. Mircea Iucu , Reşiţa IX.043 Rezolvaţi în mulţimea ℝ + sistemul de ecuaţii :

xy + yz + zx = 1 ⎧ ⎪ 2 2 2 ⎨1 − x + 1 − y + 1 − z = 3 ⎪⎩1 + x x 1 + y 2 1 + z 2 2

Prof. Aurel Doboşan , Lugoj

Prof. Nicolae Stăniloiu , Bocşa

61

62


o. ro

Clasa a X-a

X.042 Să se arate că dacă z ∈ ℂ şi z < 1 , z + 1 < 1 , atunci

X. 034 Demonstraţi că pentru orice x ≥ 0 şi n ∈ ℕ , n ≥ 2 , este

z3 +1 < 4 .

(2 + x) k ≥ 2n − 1 adevărată inegalitatea : ∑ 2 kx + k =1 n

Prof. Dorin Mărghidanu , Corabia X.035 Determinaţi numerele naturale n pentru care dezvoltarea ( 2 + 3 5 ) n conţine exact 5 termeni raţionali. *** sumei S =

3

x + y . 2

3

2

Prof. Lucian Dragomir , Oţelu-Roşu X.037 Dacă a , b , c sunt numere reale strict pozitive cu a + b + c = 1 , demonstraţi că :

1+ a 1+ b 1+ c ⋅ ⋅ ≥8 . 1− a 1− b 1− c

XI. 034 Dacă f : [ a , b ] → ℝ este o funcţie derivabilă cu proprietatea că

f( a ) = 0 , demonstraţi că există c ∈ ( a , b ) astfel încât f (c) = (b − c) ⋅ f / (c) . Prof. Dorin Mărghidanu , Corabia XI.035 Fie f : ℝ → ℝ o funcţie continuă , mărginită , cu f (0) ≠ 0 .

Arătaţi că există x1 , x 2 ∈ ℝ , x1 ≠ x 2 , astfel încât :

x1 ⋅ f ( x 2 ) + x 2 ⋅ f ( x1 ) = 0 .

eu

Gazeta Matematică X.038 Demonstraţi că în scrierea numărului ( 2501 + 50)1981 ca număr zecimal primele 3962 cifre după virgulă sunt zerouri. Prof. D.M.Bătineţu-Giurgiu , Bucureşti X.039 Fie f : A → B . Arătaţi că f este injectivă dacă şi numai dacă (∀) X , Y ⊂ A cu f ( X ) ⊂ f (Y ) ⇒ X ⊂ Y . ***

w .n

X.040 Să se determine funcţia f : ℕ → ℕ care satisface : f ( x + y ) = f ( x) ⋅ f ( y ) , ∀ x , y ∈ ℕ şi f (2006) = 3 2006 Prof. D.M.Bătineţu-Giurgiu , Bucureşti X.041 a ) Calculaţi suma S =

n

∑ k ⋅C k =1 n

k

n

, n ∈ ℕ* ;

b ) Demonstraţi că : n ⋅ 2 ≥ ( n + 1)(2 n − 1) , ∀ n ≥ 1 ;

w

c ) Demonstraţi că : n ⋅ 3 n ≥ 2 n ⋅ (2 n − 1) , ∀ n ≥ 1 . Cosmin Istodor , elev , Oţelu-Roşu

63

x

Clasa a XI-a

tri n

X.036 Dacă x , y ∈ ℝ+ şi x + y = 2 , determinaţi valoarea maximă a

X.043 Rezolvaţi ecuaţia :

Prof. Aurel Doboşan , Lugoj 136 + 225 = 64 x + 255 x . Prof. Aurel Doboşan , Lugoj x

Prof. Lucian Dragomir , Oţelu - Roşu

XI.035 Dacă f ℝ → ℝ este o funcţie derivabilă şi cu derivata funcţie periodică , arătaţi că f se poate scrie ca o sumă dintre o funcţie periodică şi o funcţie de gradul întâi. Prof.Dorin Mărghidanu , Corabia

( {

XI.036 Calculaţi lim arctg log 2 n →∞

}) , unde {x} reprezintă

n2 + n + 1

partea fracţionară a lui x. Prof. Nicoleta Bran , Craiova XI. 037 Fie f ℝ → ℝ o funcţie care are proprietatea lui Darboux şi care

⎛ f ( x) ⎞ ⎟ = x , ∀ x > 0 . Demonstraţi că f este continuă . ⎝ x ⎠

satisface f ⎜

Prof. Ionuţ Ivănescu , Craiova XI.038 Determinaţi funcţiile continue f : ℝ → ℝ care satisfac :

x y 2 f ( xy ) ⋅ f ( y ) + ⋅ f ( x) = ,∀x,y∈ℝ*. y x xy

Prof. Lucian Dragomir , Oţelu-Roşu

64


1 rămân ? 3n

n →∞

XI.041 a ) Arătaţi că pentru orice matrice A , B ∈ M 2 ( ℝ ) are loc relaţia : det( A + B) + det( A − B ) = 2(det A + det B) ;

b ) Dacă X , Y ∈ M 2 ( ℝ ) astfel încât

1 , det( X + Y ) + det( XY + YX ) = 2 atunci det( X + Y ) + det( X − Y ) ≤ 2 . Prof. Marius Damian , Brăila

b

f ( x) ∫a f ( x) + f (a + b − x) dx = ∫a f ( x)dx , arătaţi că există c ∈ ( a , b ) astfel încât (a + b) ⋅ f (c) = c . Prof. Dumitru Bătineţu-Giurgiu , Bucureşti XII.035 Fie ( G,⋅ ) un grup şi a , b ∈ G , a, b ≠ e , a , b distincte . Dacă

Prof. Viorel Cornea , Dan Ştefan Marinescu , Hunedoara

XII.037 Fie G un grup multiplicativ şi a , b ∈ G astfel încât a 10 = b10 şi a 17 = b17 . Demonstraţi că a = b . Prof. Aurel Doboşan , Lugoj XII.038 Arătaţi că există q ∈ ℚ astfel încât : π

n

w .n

XI.043 a) Calculaţi sin

12

e

2

1

0

sin x ∫ arcsin(ln x)dx + ∫ e dx = q ⋅ π ⋅ e .

⎛x ⎞ 2 5 x n +1 = 13x n + 12 ⋅ x n − 4 , ∀ n ≥ 1 . Calculaţi lim⎜ nn ⎟ . n →∞ 5 ⎝ ⎠

π

Prof. Romanţa şi Ioan Ghiţă , Blaj

2 4 2n + sin + ... + sin sin 2n 2n 2n XII.036 Calculaţi lim n →∞ 3 2n − 1 1 + sin + ... + sin sin 2n 2n 2n

eu

2

26 XI.042 Se consideră şirul definit prin x1 = , 5

b

ab 2 = b 4 şi b 5 = a 2 ,determinaţi ordinele elementelor a şi b .

b) Dacă notăm cu S ( n ) aria totală a pătratelor eliminate , calculaţi lim S (n) . Concurs Ungaria

2

XII. 034 Fie a , b > 0 , a < b şi funcţia continuă f : [ a , b ] → ℝ+*. Dacă

tri n

a ) Câte pătrate cu latura de lungime

o. ro Clasa a XII-a

XI.039 Determinaţi n ∈ ℕ * şi matricele inversabile A ∈ Mn(ℂ) pentru care ( A* ) * = A* , unde A * este adjuncta matricei A. Prof. Romanţa şi Ioan Ghiţă , Blaj XI.040 Se consideră un pătrat cu latura de lungime 1 . Ducem câte două drepte paralele la laturile pătratului şi acesta se împarte în 9 pătrate congruente;eliminăm pătratul din mijloc.Cele 8 pătrate rămase le împărţim în mod analog în câte 9 pătrate şi din nou din fiecare eliminăm pătrăţelul din mijloc ,etc. Repetăm procedeul de n ori.

Prof. Aurel Doboşan , Lugoj

Prof. Aurel Doboşan , Lugoj

;

⎛ 1 − 3⎞ ⎛ 3 ⎟, Y = ⎜ b) Fiind date matricele X = ⎜ ⎜ 3 ⎜ −1 1 ⎟⎠ ⎝ ⎝ 2n *. (X + Y ) , n ∈ ℕ

1 ⎞ ⎟ ,calculaţi 3 ⎟⎠

w

Prof. Aurel Doboşan , Lugoj

65

XII.039 Care dintre următoarele numere este mai mare : 3

2

−2

−3

a = ∫ arctgxdx sau b = ∫ arctgxdx ? Gazeta Matematică XII. 040 Există funcţii f : ℝ → ℝ continue care admit o primitivă F pentru care f ( F ( x)) + x 3 + x = 0 , ∀ x ∈ ℝ ? Prof. Antoanela Buzescu , Caransebeş

66


o. ro

ex ⎛ π⎞ dx , x ∈ ⎜ 0, ⎟ 2 tg x ⋅ sin x ⎝ 2⎠ Prof. Nicolae Dragomir , Reşiţa , Prof. Carmen Dragomir,Timişoara XII. 042 Se consideră mulţimea de matrice M = ⎧⎛ 2 x + 1 0 ⎫ x ⎞ ⎟ 1⎪ 1 ⎪⎜ 0 0 ⎟ / x ∈ ℜ, x ≠ ⎬ şi notăm G = ℝ ∖ { } . ⎨⎜ 0 5⎪ 5 ⎪⎜ 6 x 0 3 x + 1⎟⎠ ⎩⎝ ⎭ Dacă pentru orice x , y ∈ ℝ notăm x D y = x + y + 5 xy , arătaţi că M are o structură de grup în raport cu înmulţirea matricelor , izomorf cu grupul ( G , D ) . Prof. Nicolae Dragomir , Prof. Tudor Deaconu , Reşiţa XII.043 Fie (G,⋅) un grup multiplicativ având elementul neutru e . Arătaţi că dacă există un morfism injectiv f : G → G pentru care avem f ( f ( x)) ⋅ f ( x) = e , ∀ x ∈ G , atunci G este abelian . Prof. Lucian Dragomir ,Oţelu-Roşu

Rubrica rezolvitorilor

XII.041 Calculaţi : I = ∫

– punctaje realizate pentru soluţiile problemelor din RMCS nr. 15 ( în paranteză apare punctajul total realizat pentru concurs ) Clasa a III-a :

w

w .n

eu

tri n

Liceul Hercules Băile Herculane ( înv. Floarea Kuszai ) : Şandru Ilie Daniel 73 , Dobreanu Răzvan 69 , Lozovanu Dumitru 100 , Croitoru Ioana Sabina 95 ; (înv.Doina Zah ) T őr ők Bogdan 100 , Daniel Coman 85 , Mihart Georgiana 100 , Dancău Anca 100 , Dimcea AnaMaria-Alexandra 100 , Gherghina Liviu-Nicu 88 , Ferescu Liana-Maria 95 , Domilescu Manuel-Ilie 92 Şcoala nr. 1 Oraviţa (înv. Merima Velcotă ) : Pîrvu Ancuţa Iulia 67 Liceul Pedagogic C.D.Loga Caransebeş ( înv. Monica Pelin , Monica Urban ) : Lala Timotei 70 , Iordache Andreea-Claudia 100 , Băzăvan Cătălina 70 , Băzăvan Răzvan 70 , Bojin Călina Milica 67 Liceul Traian Doda Caransebeş ( înv. Marinela Galescu ) : Dragomir Ioana-Ştefania 97 , Mura Ana-Maria 82 , Moraru Dragoş 50 , Suru Alexandru Răzvan 86 , Coste Anastasia 97 , Voicu Vlad 97 Clasa a IV-a : Şcoala Bănia ( înv. Cristian Pirtea ) : Odobaşa Daniel 93 Liceul Pedagogic C.D.Loga Caransebeş ( înv. Pelin Monica , Monica Urban ) :Leon Natalia-Emilia 60 , (inst. Mirela Tătar ) : Pantiş Antonia 100 , Şerbescu Andreea 78 , Neuman Liviu 98 , Cernescu Sebastian 100 , Pop Silvia 115 , Liceul Traian Doda Caransebeş ( înv. Elena Cîrstea ) : Szabo Ildiko 95 Şcoala Generală 2 Reşiţa ( inst. Ozana Săcărin , înv. Ana Modoran , înv. Georgeta Gaiţă , înv.Mărioara Popescu ) : Bîtea Flavius 20 , Dieaconu Estera 43 , Saşec Ion Cosmin 26 , Bălu Lorena 20 , Baierle Amalia 30 , Izvernar Daniel Otniel 27 , Banţa Nicolae Alex 20 , Barbu Bogdan 98 , Ungurean Simona Roxana 28 , Wettori Michael Sebastian 100 , Balasan Roberta Andreea 65 , Rogge Petra-Ana 100 , Ţeudan Adina 100 , Drăghici Livia-Liliana 100 , Tomescu AlinaNicoleta 100 ,Uzoni Ribana-Alexandra 100 , Dăescu Andrei 100 ,Brebenariu Octavia-Patricia 100 , Mregea Natalia Patricia 100 , Blaga Georgiana 100 , Popa Andreea 100 , Onofrei Iulia 70 , Borchescu Daiana Maria 39 , Zăria Gergiana 60 , Bolfă Larisa 30 , Aghescu Monica Larisa 100 , Darie Mădălina Mihaela 70 , Lungu Cosmin 30 , Ţoţu

67

68


o. ro

tri n

Grup Şcolar Industrial Oţelu-Roşu ( prof.Adriana Dragomir) : Dumitresc Cecilia Graţiela 143, Albai Cosmin 72 , Dragomir Claudiu 63 , Nasta Laura 145 , Gemănariu Trăienică 56 . Şcoala Generală nr.3 Oţelu-Roşu (prof.Boldea Felicia ) : Buzuriu George 113 Şcoala Rusca-Teregova (prof.Ciucă Sorin ) : Stepanescu Georgeta Mihaela- ( 18), Codoşpan Florinela 104,Milu Ionela 19,Banda Traian Dani 75, Humiţa Maria 44 ,Blaj Marinela Alisa 96 , Berzescu Nicolae 22 , Davidescu Toma 38 , Curmei Roxana Andreea 24 , Şcoala Vîrciorova ( prof. Ioan Liuba ) : Măran Marius 108 Fără menţionarea şcolii şi profesorului ! : Dumitraşcu Andreea , Caransebeş 148 Clasa a VI-a

Grup Şcolar Anina ( prof. Petrişor Neagoe ) : Gîrjan Oana Nicoleta 10 , Radu Bianca 10 , Şcoala nr. 1 Anina ( prof. Marin Constantin Cleşiu ) : Borcean Andreas 38 , Juraszic Claudia 40 . Şcoala Bozovici ( prof. Maria Bololoi) : Borchescu Ana Maria 70 , Petre Estera Alina 70 , Borozan Florina Elisaveta 89 Liceul Traian Doda Caransebeş (prof.Delia Dragomir): Szabo Cristian 69 , Mocanu Ioana Dora 77, Peia Vigia Alexandra 50 Liceul Pedagogic C.D.Loga Caransebeş(prof.Dorina Humiţa): Pleşko Cosmin Peter 28(28) , Semenescu Anca 160 ,Borcean Gheorghe 63 ,Bob Cristiana 57 ; (prof. Mariţa Mirulescu), Todorovici Lucian 45 , Vladu Cristian 76 , Matei Sergiu 30 Şcoala nr.2 Caransebeş ( prof. Corîci Carina ): Antoce Alexandru 97 Colegiul Naţional Carol I Craiova ( prof. Monica Stanca ) Stanciu Ioan 113 ( mare dezordine , încearcă să respecţi regulile amintite ; în rest , felicitări ) Liceul de Artă Reşiţa (prof.Adriana Mara ) :Goicovici Denisa 72 Şcoala Generală 9Reşiţa ( prof. Irina Avramescu ) : Kormos Nicholas 33 Şcoala Rusca-Teregova (prof.Ciucă Sorin ):Paşan Petru 126 , Linţu Florin Cosmin 64 ,Blaj Ilie Dănuţ 68 , Vernicuţa Petronela 63 , Stepanescu Elisabeta 34 , Banda Vasile 36 , Banda Ionela Mitra 43 , Dumitrică Eva Daniela 31 , Stancu Ana Maria 9 , Berzescu Maria 32 , Humiţa Ana 15 , Stan Stana 8 Şcoala Generală nr. 1 Oţelu-Roşu ( prof.Heidi Feil , Cecon Iulia ): Duma Andrei 133 , Bistrian Florina 55 , Ivu Nicoleta 65 .

w

w .n

eu

Mădălina 88 , Toader Alexandra Anastasia 40 , Şarga Robert 80 , Toader Teodora 40 , Nicorici Bogdan 100 , Şcoala Generală 9 Reşiţa ( înv. Adina Belu ) Peptan Alexandru-Florin 100 , Grădinaru Adelina 98 , Arusoaiei Iulian 98 , Manciu Bogdan 99 , Nedelea Adrian-Gabriel 95 , Lazăr Silviu – Ioan 120 , ( inst. Mariana Mitrică ) : Muscai Lorena 98 , Clasa a V-a : Şcoala nr.1 Anina (prof. Manuela Skopecs): Rotaru Ana-Maria 40 , Drăgilă Patricia 79 ,Vrînceanu Cezar 35 , Sârghie Bianca 56 Liceul Hercules Băile Herculane ( prof.Marius Golopenţa): Iacobici Pavelina 30 , Anton Alexandru Lucian 83 ( trimite toate problemele într-un singur plic ! ) , Tabugan Călina Dana ( 139 ) , Lolea Sandra 68 , Muică Grozăvescu Mihaela 48 ,Popeangă Raluca Ştefania 132 , Basarabă George 126 , Martin Patricia 17 ,Măncescu Maria-Manuela 28 , Cernea Alexandra 15 , Şcoala Bozovici (prof.Iosif Găină ) : ,Bratosin Felix 65,Barbeş Cezara 55 , Păunescu Alexandra 55 ,Băin Daiana 25 , Băcilă Cristiana 67 Şcoala nr.2 Caransebeş ( prof. Corîci Carina ): Agape Oana Gabriela 174 , Bădăluţă Alexandru 96 , Bărbuceanu Florina 122 , Liceul Pedagogic C.D.Loga Caransebeş ( prof.Mariţa Mirulescu) Sâsâeac Iulia Irina 74 , Tătar Octavian 135 , Ion Răzvan 120 , Vela Silviu 66 , Antonescu Ionica Nicoleta 38 , Timofte Tina 75 Liceul Traian Doda Caransebeş (prof.Adrian Dragomir ): Stoicănescu Gelu 165 , Rada Cristiana 157 , Keleti Edith 143 , Stepanescu Mihai 80 , Burciu Daniel 36 , Puşchiţă Daniel 80 , Colegiul Naţional Carol I Craiova ( prof. Monica Stanca ) Stanciu Ioan 113 ( mare dezordine , încearcă să respecţi regulile amintite ; în rest , felicitări ) Şcoala nr.1 Moldova-Nouă ( prof. Marioara Radosavlevici ) Gîrjan Laura Nicoleta 74 , Craiovan Andreia Dana 77 Şcoala Ciclova Română (prof. Geta Mîşcoi ) Bănuc Vasilică – Angel 40 Şcoala Generală 2 Reşiţa (prof.Mariana Drăghici): Meşter Amalia 87 , Mihăilă Flavius 41 , Moţ Daria 46 , Cernea Serena 57 , Scutaru Lavinia 15 , Radcu Antonia 50 , Moldovanu Alina 33 , Irina Ciorogar 67 , Pascu Andra Diana 119 Şcoala nr. 1 Oraviţa ( prof. Camelia Pîrvu ) : Pelian-Popa Ioana 29

69

70


o. ro

tri n

Clasa a IX-a Liceul Hercules Băile Herculane ( prof.Marius Golopenţa): Feneşan Manuela 66 , Caraiman Gabriela Sofica 66 Liceul Teoretic Eftimie Murgu Bozovici (prof.George Pascariu ): Şuveţi Pavel 9 . Liceul Pedagogic C.D.Loga Caransebeş(prof.Lavinia Moatăr ): Kremer Emanuela 83 , Gurgu Caius 64 , Iliescu Marcel 28 . Liceul General Dragalina Oraviţa (prof. Mihai Lazarov ) : Nezbeda Harald 42 , Răşinariu Lucian 80 Grup Şcolar Industrial Oţelu-Roşu ( prof.Lucian Dragomir ): Unguraş Dragoş 106 , Dragomir Lucia 45 , Buzuriu Alina 52 , Popa Roxana 45 , Muntean Cristian 45 .

Clasa a X-a Liceul Pedagogic C.D.Loga Caransebeş ( prof.Mariţa Mirulescu): Labo Laurenţiu 45 ,Roată Ramona 21 , Mărgan Larisa 23 , Munteanu Laura Loredana 35 , Stănilescu Maria 29 , Colţan Anca 26 , Bămescu Monica 23 , Cornean Cristian 25 , Beja Ancuţa 36 , Ciortan Oana 35 , Ionescu Alin 25 Liceul Traian Doda Caransebeş (prof.Lavinia Moatăr ) : Zoican Andrei 23 , Voinea Alexandra 75 , Cărăbaş Florentina Angela 68 , Dochin Luminiţa 68 , Mutuleanu Alexandra 57 , Cuţitoi Simina 96 , Petruş Laura 100 , Aghescu Loredana 73 , Guţulescu Oana 91 , Burghelea Bogdan 43 , Piele Ionuţ 48 ( prof. Delia Dragomir ) : Beldie Anca 58 , Iacob Alexandra 70 , Frăţilă Alina-Alexandra 73 Grup Şcolar Industrial Oţelu-Roşu ( prof.Lucian Dragomir ): Istodor Cosmin 54 , Ciobanu Constantin 42. Clasa a XI-a

w

w .n

eu

Clasa a VII-a Şcoala nr.1 Anina (prof. Manuela Skopecs): Golîmba Pavelina-Adelina 20 , Cleşiu Marian Cătălin 60 , Tatar Santra Sorina 16 Şcoala nr.2Anina ( prof. Nicolae Seracin ) : Busa Bianca 10 Şcoala Rusca-Teregova (prof.Ciucă Sorin ) :, Humiţa Toma 29 , Iciu Gheorghe 35 , Stepanescu Mihai 27 , Raduia Ştefan 31 , Gherga IonuţBarbu 40 , Banda Anca 23 , Stepanescu Ianăş 32 , Moacă Ion 16 , Rădoi Georgeta 42 , Gherga Petru 29 , Popa Petru-Ionuţ 42 , Banda Iosif 21 ,Dumitrică Octavian 17 (prof.Ilie Damian ) : Ciucă Cristian Sorin 68 Liceul Hercules Băile Herculane ( prof. Constantin Bolbotină ) : Păleanu Oana Georgiana 10 ( multe probleme din RMCS 14 , care însă au fost rezolvate în RMCS 15 ! ) , Dimcea Ion-Cristian 97 Şcoala Berzovia ( prof. Dan Miholcea ) : Chisăliţă Alexandra 40 Liceul Traian Doda Caransebeş (prof.Delia Dragomir): Novăcescu Dorin 85 , Zanfir Cristian 60 , Vid Cristina 70 , Baneu Petru 80 Liceul Pedagogic C.D.Loga Caransebeş(prof.Diana Hurduzeu ) : Prunar Victor 210 . Şcoala Generală 2 Reşiţa (prof.Marius Şandru): Meşter Sergiu 30 Liceul de Artă Reşiţa (prof.Adriana Mara ) : Cherloabă Edith 77 Şcoala Generală nr. 1 Oţelu-Roşu (Prof. Heidi Feil ) : Atinge Carina 107 , Cococeanu Oana Maria 125 Şcoala Generală nr. 3 Oţelu-Roşu ( prof.Felicia Boldea ): Ştefănigă Sebastian , Lazăr Dinu 58 , Silianovici Alin 46 , Bărângă Sergiu 69 Clasa a VIII-a Şcoala Rusca-Teregova (prof.Ciucă Sorin) : Codoşpan George 48 , Humiţa Maria – Mirabela 26 , Gherga Patricia 26 , Stepanescu AncaLiliana 25 , Stepanescu Adamescu Ioan 32 , Humiţa Elisabeta 33 , Cobel Ştefania Ionela 25 , Gherga Constantin 30 , Banda Ioan 12 , Banda Maria 25 , Gherga-Blaj Elisabeta-Ionica 24 Şcoala Generală nr. 2 Bocşa (prof. Veronica Todor ) : Stăniloiu Ovidiu 155 . Liceul Pedagogic C.D.Loga Caransebeş(prof.Lavinia Moatăr ):Milcu Roxana 227 , Timofte Andrei 68 , Cristescu Loga Cella 60 , Moatăr Alexandra 80 , Vlad Adina 189 , Megan Ligia 55 , Ploştinaru Diana 40

71

Liceul Teoretic Traian Doda Caransebeş (prof.Lavinia Moatăr):Enache Bianca Emilia 36 , Mureşan Viorel Dan 40 , Gherghinuţă Florin 40 Clasa a XII-a Liceul Teoretic Traian Doda Caransebeş (prof.Iacob Didraga ) Creac Larisa-Anca 30 Grup Şcolar Industrial Oţelu-Roşu ( prof.Lucian Dragomir ): Morariu Claudia 86, Teişanu Iuliana 20, Bălan Carmen 20, Sandu Ionela 30 , Agliceriu Andreea 30

72


o. ro

7.Ruva Gheorghe 8.Skopecz Manuela 9.Seracin Nicolae 10.Burciu Simona 11.Oravitan Florica 12.Dancea Ioan

eu

23. Mustata Maria 24. Peter Eva Maria 25.Staicu Lenuta 26. Seracin Ioan 27. Staniloiu Manuela 28.Staniloiu Nicolae 29. Todor Veronica 30. Todor Ioan 31. Zsibriczi Ecaterina 32. Roos Carol 39.Opranescu Angela 40.Roman Simion 41.Tatucu Anton 42.Vasile Mihaela 43.Golopenta Marius

w .n

Zona Anina 1.Clesiu Marian 2.Ichim Dumitru 3.Lungulescu Petru 4. Neagoe Petrisor 5. Neagoe Nicoleta 6.Pruteanu Silvia Zona Bocsa 13.Almajan Catalin 14.Boriuc Veturia 15.Branzei Monica 16. Ciolos Aurelia 17. Costa Veronica 18.Iatan Rodica 19. Lupulescu Daniela 20. Lungu Aurel 21. Lungu Emilia 22. Miholcea Dan Zona Baile Herculane 33.Bolbotina Constantin 34.Boscota Calina 35.Haracicu Maria 36.Horescu Ion 37.Ienea Maria 38.Lalescu Tiberiu Zona Bozovici 44.Borchescu Marius 45.Gaina Iosif 46.Gaina Nicolae Zona Caransebes 50.Bistrian Ana 51.Buzescu Antoanela 52.Ciocan Florin 53.Ciuca Sorin 54.Corici Carina

55.Corici Sebastian 56.Dragota Ana 57.Didraga Iacob 58.Didraga Elena 59.Dragomir Petru 60.Dragomir Gorici 61.Dragomir Delia 62.Dragomir Adrian 63.Hurduzeu Diana 64.Humita Dorina Zona Moldova Noua 80.Albeanu Vasile 81.Cristescu Fanica 82.Damian Lacrimioara 83.Gadea Vasilica 84.Hergane Adam 85.Huza Vasile 86.Mihart Nicolae 87.Murg Stana 88.Mateescu Elena 89.Panici Nadita Zona Reşiţa (gimnaziu) 99.Coanda Camelia 100.Radoi Mirela 101.Curescu Simona 102.Tunea Aurica Lica 103.Sandru Marius 104.Draghici Mariana 105.Ciulu Loreta 106.Bihoi Georgeta 107.Buzila Claudia 108.Motco Monica 109.Avramescu Irina Zona Oravita 121.Iacob Ionel 122.Pricoiu Carmen 123.Miscoi Geta

tri n

Membrii Filialei Caraş-Severin a Societăţii de Ştiinţe Matematice din România (cu cotizaţia plătită pe 2006)

47.Pascariu George 48.Rincu Pavel 49.Suta Floare

w

65.Gherga Petru 66.Ivascu Nicoleta 67.Isac Simion 68. Lucaci Marinela 69.Liuba Petru

73

70.Moatar Lavinia 71.Mirulescu Marita 72.Miuta Bocicariu Janet 73.Popescu Constanta 74.Sandutu Janeta 75.Stavaroiu Eugen 76.Susoi Paul 77.Tamas Maria 78.Tuican Liliana 79.Vasiluta Violeta

90.Popa Ionica 91.Radosavlevici Marioara 92.Rujici Iasna 93.Schiha Emilia Dana 94.Scortan Gheorghe 95.Stoicovici Mirela 96.Vladu Dumitru 97.Vladu Sanefta 98.Voilovici Aurelia

110.Gosa Anca 111.Belci Ion 112.Chis Vasile 113.Rosu Lia-Adriana 114.Simulescu Susana 115.Untanu Giurgevica 116.Andras Nicolae 117.Mara Adriana 118.Deaconu Tudor 119.Socol Maria 120.Dicu Lenuta 128.Parvu Camelia 129.Curea Nicolae 130.Imbri Nicolae

74


o. ro

144.Czanczinger Erica 145.Albai Camelia 146.Sarbu Eftimie 147.Ferdean Arjentia 148.Dragomir Adriana 149.Dragomir Lucian 150.Frent Dana

159.Ghimboaşă Pavel 160.Bejan Otilia 161.Mihailovici Dana 162.Buzilă Mircea 163.Călin Ciprian 164.Deaconu Dorel 165.Bădescu Ovidiu

pentru editarea revistei noastre şi pentru recompensarea elevilor care merită (aşa s-a născut ideea concursului RMCS). De fapt , o să-mi permit să citez din finalul hotărârii luate la întâlnirea din 1984 de fondatorii Gazetei Matematice: “… să căutăm zece personae care să vrea să dea sumele necesare pentru apariţia revistei în primul an, iar toate veniturile acelui an să se capitalizeze, pentru a se garanta publicarea revistei în anii următori, când unul sau mai mulţi din cei zece nu ar voi să mai dea sumele necesare … “ . Aşadar o muncă şi un efort voluntare pentru viitorul matematicii din România; sperăm să retreziţi interesul pentru Gazeta Matematică, să ţineţi treaz interesul pentru RMCS, pentru viitorul matematicii în Caraş-Severin, sperăm să vă întrebaţi mai mult ce faceţi pentru ţara de mâine, nu ce face ea azi pentru voi . Lucian Dragomir, cu voia Dumneavoastră, actual preşedinte al filialei SSMR

eu

Zona Otelu-Rosu 136.Ardelean Ion 137.Florea Viorel 138.Popa Amalia 139.Cecon Iulia 140.Feil Heidi 141.Iuhasz Cornelia 142.Boldea Felicia 143.Jurj Nutu Zona Resita (liceu) 151.Tunea Ana 152.Balan Gheorghe 153.Mateia Monica 154.Avram Matei 155.Surugiu Gabriela 156.Farkaş Viorica 157.Pop Cristiana 158.Stanciu Ana

131.Ticu Maria 132.Lazarov Aurica 133.Lazarov Mihael 134.Costea Mihaela 135.Pistrila Ion

tri n

124.Iancu Maria 125.Draghicescu Tomita 126.OtimanRazvan 127.Badoi Marian

w

w .n

Trebuie să mărturisesc, cu puţină dezamăgire în suflet, că ne aşteptam la mai mult: avem în judeţ peste 300 de profesori şi doar cei de mai sus sunt membrii cotizanţi ai SSMR; revista noastră chiar apare într-un tiraj foarte mic, majoritatea celor care o achiziţionează fiind elevi (de fapt, poate că lor ne adresăm în primul rând , în încercarea de a face viitorul mai bun , în speranţa că ei vor schimba ceea ce nu suntem noi în stare). Nu mai vorbim de Gazeta Matematică –avem sub 30 de abonamente în judeţ , incredibil . De ce să fim membrii ai SSMR , de ce să contribuim financiar , anual, cu o sumă (modestă de fapt ) la fondul Societăţii. Ce ne adduce ea? Mai explicăm o dată ce se întâmplă cu aceşti bani: jumătate sunt depuşi în contul SSMR la sediul central din Bucureşti (scop principal: editarea Gazetei Matematice, care în mare măsură aşa a reuşit să “trăiască”110 ani), jumătate rămân la noi, în judeţ, 75

76


Issuu converts static files into: digital portfolios, online yearbooks, online catalogs, digital photo albums and more. Sign up and create your flipbook.